MBE QUESTIONS

Réussis tes devoirs et examens dès maintenant avec Quizwiz!

A woman filed a negligence suit against her friend's sister after the woman, the friend, and the friend's sister accidentally started an electrical fire in the woman's home while they were building a lamp together. At trial, it was determined that the electrical fire was due to multiple causes: (i) the sister's faulty wiring of the lamp, (ii) the woman's installation of a light bulb with a wattage too high for the lamp, and (iii) the friend's choice of a highly flammable material to make the lampshade. At the conclusion of the trial, the jury determined that the woman suffered $200,000 in damages, and apportioned fault as follows: the woman was 20% at fault, the friend was 30% at fault, and the friend's sister was 50% at fault. What is the maximum amount that the woman can recover from the friend's sister?

$160,000. Answer choice A is correct. In jurisdictions that have adopted the doctrine of pure comparative negligence (which is the default on the MBE, unless you are told otherwise), a plaintiff's contributory negligence is not a complete bar to recovery. Instead, the plaintiff's full damages are calculated by the trier of fact and then reduced by the proportion that the plaintiff's fault bears to the total harm. Here, because the jury has determined the woman to be 20% at fault for the electrical fire, her total recovery will be reduced by that percentage. Under the doctrine of joint and several liability (which is also the default on the MBE, unless you are told otherwise), each of two or more defendants who is found liable for a single and indivisible harm to the plaintiff is subject to liability to the plaintiff for the entire harm. The plaintiff has the choice of collecting the entire judgment from one defendant, the entire judgment from another defendant, or portions of the judgment from various defendants, as long as the plaintiff's entire recovery does not exceed the amount of the judgment. Here, the friend and the friend's sister are joint tortfeasors and are both jointly liable to the woman. Thus, the woman can recover the entire amount, less the 20% attributable for her own negligence, $160,000 ($200,000 minus $40,000) from either of them. Answer choice B is incorrect because it incorrectly subtracts the friend's proportionate fault from the total amount of damages ($200,000 minus $60,000 equals $140,000), not the woman's fault, which is necessary to determine the total amount of damages the woman can recover. Answer choice C is incorrect because it incorrectly accounts for the sister's proportionate fault ($200,000 minus $100,000 equals $100,000). Answer choice D is incorrect because it incorrectly accounts for the sister's and the woman's proportionate fault ($200,000 minus $100,000 and $40,000 equals $60,000), instead of just the woman's fault, in determining the maximum possible recovery by the woman.

A state official was charged with two federal offenses—bribery in connection with a program administered by the state official that receives federal funds, and interstate travel for a criminal purpose. The bribe allegedly consisted of paying the expenses of an out-of-state trip taken by the state official in exchange for awarding a grant under the federal program to the payor of the expenses. The jury found the state official guilty of bribery but not guilty of the offense of interstate travel for a criminal purpose. The state official has challenged his conviction for bribery as unconstitutional. How should the court rule on this challenge?

Against the state official, because acquittal by a jury of an offense does not mandate acquittal of the predicate offense. Answer choice C is correct. When a jury renders a verdict that a defendant is guilty of certain offenses but not guilty of other related offenses, the guilty verdict is not reviewable on the grounds of inconsistency, even when the jury acquits the defendant of an offense that is a predicated on an offense for which the same jury finds the defendant guilty. Consequently, although the defendant was acquitted by the jury of the travel offense which was predicated on bribery as its criminal purpose, the jury's verdict finding the defendant guilty of bribery is not reviewable on inconsistency grounds. Answer choice A is incorrect because, when there are inconsistent jury verdicts, due process does not mandate the overturning of the guilty verdict on the grounds that it is inconsistent with the acquittal. Answer choice B is incorrect because one of the offenses is not a lesser-included offense of the other offense; each requires proof of an element that the other does not. The travel offense requires proof that the travel involved interstate travel, while the bribery offense requires the solicitation or acceptance of something of value by a state or local official. Moreover, while the Double Jeopardy Clause may prevent a defendant from being punished for both a lesser- and a greater-included offense, it does not prevent a defendant from being tried for both offenses in the same trial, when the defendant is punished for only one of them. Here, the state official was only convicted of the bribery offense. Answer choice D is incorrect because a defendant may challenge a jury verdict on appeal on a number of grounds, including insufficiency of the evidence.

A man had spent the afternoon at the beach with his girlfriend; each of them had consumed a significant quantity of alcohol. The man saw one of the town's wealthiest residents arrive at the beach, spread out a beach towel, put down a large cloth bag on the towel, drop what looked like a wallet into the bag, and run into the ocean. The man recounted his observations to his girlfriend regarding the wealthy man's actions. She didn't respond, but walked over to the bag and opened it. The bag did not contain the man's wallet, but it did contain an expensive ring. She took the ring, and closed the bag. The wealthy man's companion, who was just coming onto the beach, observed the girlfriend's action. She was arrested. Soon thereafter, the man was arrested as well. The man is charged, among other crimes, with conspiracy to commit larceny. The applicable jurisdiction has adopted a conspiracy statute based on the Model Penal Code. Which of the following is the weakest argument that the man can advance in defense of the charge?

Answer choice C is correct. Factual impossibility is not a defense to conspiracy. Answer choice A is incorrect because, even though the jurisdiction has adopted a unilateral approach to conspiracy, in order for a defendant to be convicted of conspiracy, the defendant must agree to commit an unlawful act even if the other party does not. Here, the man's mere recounting of his observations would likely be insufficient to establish such agreement. A conspirator can be convicted of both the offense of conspiracy and all substantive crimes committed by any other co-conspirators acting in furtherance of the conspiracy. Answer choice B is incorrect because, here, the man could argue that the theft of the ring was an independent crime, and was not in furtherance of the conspiracy to steal the man's wallet. Answer choice D is incorrect because conspiracy is a specific intent crime for which voluntary intoxication may be a defense

A wealthy mother disapproved of her daughter's boyfriend, a mechanic in a small auto shop. Despite her protestations, the daughter refused to break up with him, and the mother decided to get rid of the boyfriend before it was too late. She stole a key to the boyfriend's shop from his keyring, gave the key to her husband, and told her husband that she wanted the boyfriend dead. The husband formulated a plan to kill the boyfriend but to make it look like an accident by sneaking into the auto shop in the middle of the night and rigging a car that the boyfriend was working on to fall on top of him. The mother had a change of heart and told her husband that she no longer wanted to go through with the plan; however, the husband sneaked into the shop and rigged the car later that night. The boyfriend went to work the next day, and the rigged car fell on him and crushed him to death. The husband was arrested for first-degree murder due to a hidden security camera in the garage. He subsequently exposed his wife's participation in the plan, and she was arrested as well. A state statute defines first-degree murder as premeditated and deliberate. The jurisdiction follows the Model Penal Code. What is the most serious crime, listed below in descending order of seriousness, for which the mother can be convicted?

CONSPIRACY. A conspiracy is an agreement to accomplish an unlawful purpose plus the intent to accomplish that purpose. The Model Penal Code (MPC) and the majority rule require a legal or illegal overt act in furtherance of the conspiracy. Here, wife's stealing the key to the boyfriend's shop and giving it to her husband is an overt act in furtherance of the conspiracy. The MPC allows subsequent withdrawal by a co-conspirator after the overt act, but the defendant must thwart the performance of the planned crimes. The wife told the husband that she no longer wanted to go through with killing the boyfriend, but she did not thwart the plan so she is guilty of conspiracy under the MPC. Answer choice A is incorrect because although the wife was unable to effectively withdraw from the conspiracy, her withdrawal was effective with respect to the substantive crime of first-degree murder committed by her husband. Answer choice C is incorrect because although the wife is guilty of solicitation, she is also guilty of conspiracy, which is a more serious crime. Answer choice D is wrong because the wife is guilty of both solicitation and conspiracy.

An auction company held an auction of a rare historical document consigned to the company for sale by a private individual. At the auction, the document sold to a partnership for $50,000. Before the document was delivered to the partnership, a library contacted the auction company and asserted ownership of the document. The auction company filed a federal statutory interpleader action in federal district court, naming the partnership, the individual partners, and the library as defendants. State law permits a partnership to be sued as a separate entity from its individual partners. The forum court is located in the district where the auction company has its principal place of business and in the state in which the auction company is incorporated. The partnership consists of two partners. Partner A, the partner who bid on behalf of the partnership at the auction, is domiciled in another district of the same state in which the forum court is located. Partner B is domiciled in a neighboring state and has no personal contacts with the state in which the forum court is located. The library is domiciled in a third state. Partner B has filed a motion to dismiss the action based on lack of subject matter jurisdiction, personal jurisdiction, and improper venue. How should the court rule on this motion?

Deny it, because the court has both subject matter and personal jurisdiction and venue is proper. As discussed in detail below with regard to the incorrect answer choices, the court has both subject matter jurisdiction and personal jurisdiction and venue is proper. Answer choice A is incorrect. It is true that this action does not satisfy the requirements for a general diversity-based action. The amount-in-controversy ($50,000) does not exceed the $75,000 threshold required for a general diversity-based action. In addition, diversity of citizenship does not exist for such an action because the plaintiff, the auction company, is a citizen of the same state as one of the partners. Although the partnership was named as a defendant, for purposes of determining subject matter jurisdiction, the partnership is considered to be a citizen of each state in which a partner is a citizen. However, because this is a federal statutory interpleader action, the value of the property at issue must only exceed $500, which is the case with this document, and any two adverse claimants must be citizens of different states. Because neither partner is domiciled in the state in which the library is domiciled, the minimum diversity required under a federal interpleader action has been satisfied. Consequently, the forum court has subject matter jurisdiction over this action. Answer choice B is incorrect. While Partner B is not domiciled in the forum state and has no personal contacts with that state, nationwide personal jurisdiction over a party exists with regard to an action brought under the federal interpleader statute. Answer choice C is incorrect. While none of the claimants reside in the district in which the forum court is located, the partnership, an entity with the capacity to sue and be sued in its common name under applicable law, is deemed to reside in any judicial district in which such defendant is subject to the court's personal jurisdiction. Because, as noted with respect to answer choice B, the federal interpleader statute provides for nationwide personal jurisdiction over a party, venue is proper in the forum court.

A plaintiff brought a tort action in diversity against two defendants based on fraud that occurred while the plaintiff was on a business trip in a foreign country with the two defendants. The plaintiff filed the action in the federal district court located in a state adjacent to the state in which the plaintiff resides. One of the defendants is domiciled in the forum state, but resides in the other federal judicial district in that state. The other defendant, who resides across the country in a third state, but was served with process in the forum state, has filed a timely motion to dismiss the action for lack of proper venue. How is the court likely to rule on this motion?

Deny the motion, because one of the defendants is domiciled in the forum state. The two most common venue rules do not apply because (1) all of the defendants do not reside in the same state, and (2) the events giving rise to the claim occurred in a foreign country. Hence, in this action, venue is proper in the judicial district in which one of the defendants is subject to personal jurisdiction. The defendant who is a domiciliary of the forum state is subject to personal jurisdiction with respect to an action brought in either federal district court located in that state. Consequently, venue is proper in the forum court. Answer choice A is incorrect because, while the convenience of the parties is a factor in determining whether to transfer an action to another court, it is not automatically the controlling rule in determining whether venue is proper. Answer choice C is incorrect for the same reason. Answer choice D is incorrect because, while one option for proper venue is the judicial district in which at least one defendant resides, that option also requires that all defendants reside in the same state in which the district is located. However, the failure to satisfy the requirements for this option is not fatal. Even when neither of the basic alternatives for proper venue is applicable, venue is proper in a judicial district in which any defendant is subject to personal jurisdiction with respect to the action.

A woman was charged with battery for trying to stab a man with a swizzle stick at a bar. During the trial, a toxicology report was admitted by the defense proving that there was no alcohol in the woman's system when she committed the act, but that there were traces of Rohypnol (i.e. "the date rape drug") in her system—the effects of which are very similar to intoxication. The defense asked for instructions that if the jury believed the woman was under the influence of Rohypnol at the time of the crime—and that she would not have committed the crime otherwise—that the jury should acquit her. What evidence would most support the defense's argument in favor of this jury instruction?

Evidence that a man talking to defendant at a bar earlier that night had put the Rohypnol in her club soda when she was in the bathroom.Involuntary intoxication would most strongly support an acquittal. Involuntary intoxication is a defense when the intoxication serves to negate an element of the crime, including general as well as specific-intent and malice crimes. To be considered involuntary, the intoxicating substance must have been taken without knowledge of the intoxicating nature of the substance, including substances taken pursuant to medical advice. If someone put the Rohypnol in the woman's drink without her knowledge, then she would have a defense to the general intent crime of battery. Answer choice B is incorrect because voluntary intoxication is not a defense to battery, and addiction does not qualify as involuntary. Answer choice C is incorrect for the similar reason that voluntary intoxication is not a defense to battery, regardless of the intoxicant's effect on a reasonable person. Answer choice D is incorrect because provocation is generally not a defense to battery.

A man placed an advertisement on an internet website offering to sell a rare coin in return for $50,000. A woman responded to the advertisement, met the man, inspected the coin, and offered to buy it. The man told her that others had expressed an interest in the coin, but that she could have it if she paid a $15,000 cash deposit within one day, with the balance paid by the end of the week. The woman came back the next day and gave the $15,000 to the man, who promised to give her the coin later that week in return for the balance of the purchase price. The man took the $15,000 and promptly flew out of state with the coin, which he never intended to sell in the first place. With which of the following crimes can the man properly be charged?

False pretenses. The crime of false pretenses requires obtaining title to the property (including money) of another person through the reliance of that person on a known false representation of a material past or present fact, when the representation is made with the intent to defraud. Here, the woman transferred title to her $15,000 to the man, relying on his false representations, which were made with the intent to defraud her. Accordingly, the man can properly be charged with the crime of false pretenses. Answer choice A is incorrect. Larceny by trick requires that the defendant obtain possession of, but not title to the property owned by another. Here, the woman intended to pass title to the money, not merely temporary possession, to the man. Accordingly, larceny by trick does not apply. Answer choice B is incorrect. Embezzlement requires fraudulent conversion of the property of another by a person who is in lawful possession of the property. Here, the man was not in lawful possession of the property, since he acquired it by fraud. Therefore, embezzlement does not apply. Answer choice D is incorrect because the man can be charged with false pretenses.

The owner of a residence contracted with a privately owned utility to provide internet service to the residence. After several years of paying the monthly bills sent by the utility, the owner failed to pay three monthly bills in a row. The contract did not specify what action the utility was required to take in such a circumstance. The utility, after giving the owner two days' advance notice, terminated the owner's access to the internet. After terminating the service, the utility provided the owner with the opportunity for a hearing on the matter. The owner has filed suit against the utility for violating of the owner's constitutional rights by failing to give her a hearing before taking such action. The owner's complaint points out the utility in question is a publicly regulated monopoly. How is the court likely to rule on the owner's lawsuit?

For the utility, because the utility is privately owned. In order for a person to successfully assert a violation of her procedural due process rights, the violation must be committed by a state actor or by a private person who is treated as a state actor (there must be "state action"). Here, although the utility is a publicly regulated monopoly, it is privately owned. Public regulation of a private entity, even an entity that has a monopoly with regard to the delivery of its product or service, is not sufficient state involvement to result in the characterization of the utility's conduct as state action. Answer choice A is incorrect. Although a contract right is a property interest that is protected by the procedural due process aspect of the Fourteenth Amendment, here the utility is not a state actor, and the state oversight of the utility is insufficient for it to be treated as a state actor. Answer choices B and D are incorrect. Although a person whose protected property or liberty interest is being deprived is entitled to notice and an opportunity to be heard, the action causing the deprivation must be undertaken by a state actor or a private person whose conduct is treated as state action. Here, the termination of internet service was effected by a private entity, not a public utility. Therefore, these procedural requirements do not apply because there is not state action.

A woman suffered from a debilitating disease, and her husband convinced her that having sexual intercourse with him would cure the disease. The husband knew that his statement was false. Relying on this statement, the woman gave her consent, and the two had sexual intercourse. Later, the woman learned that intercourse could not and did not cure her disease, and notified the police. Rape is statutorily defined as "sexual intercourse with a female against her will." The husband was convicted of rape. If the husband appeals the conviction, how should the appellate court rule on the appeal?

Fraudulent conduct does not negate consent in most situations. Here, the man induced the woman to consent to sexual intercourse through false promises--"fraud in the inducement"--but that, standing alone, does not negate consent under the common law. Here, the man made false promises to obtain consent, but did not conceal the actual nature of the act; consequently, the fraud was in the inducement, not in the factum. Answer choice B is an incorrect statement of the law; the applicable statute does not carve out immunity for persons accused of raping a spouse. Answer choice C is incorrect because, as discussed above, fraud in the inducement does not negate consent. Answer choice D is also an incorrect statement of the law; rape is a general intent crime.

A defendant in a robbery case was awaiting trial, though his attorney had negotiated with the prosecution to allow him to be under house arrest while awaiting trial, instead of sitting in jail. The police had been unable to find the gun that the defendant purportedly used to commit the robbery, and knew that finding the gun would greatly help the prosecution make its case. One of the police officers decided to visit the defendant at his home. After talking with the officer for about 30 minutes, the defendant decided to confess to the robbery. The defendant also admitted that the gun was hidden in a random office building downtown, which the police had never thought to search. Based on the confession, the officer got a warrant to search the building and found the gun. The prosecutor plans to introduce both the confession and the gun into evidence at the defendant's trial. On a motion by the defense to suppress the confession and the gun, how should the court rule?

Grant the motion as to both the confession and the gun. The evidence should be suppressed because there were both Fifth and Sixth Amendment violations. The defendant's Sixth Amendment rights were violated because he was represented by counsel in the matter at issue and there was no effective waiver of that right. Evidence obtained in violation of a defendant's Sixth Amendment rights may not be introduced at trial. This is the case for both statements of a defendant and any evidence derived from such statements. In addition, the defendant's Fifth Amendment rights were violated. Since the defendant here was under house arrest, he was in "custody." A reasonable person would not believe that he was free to leave under such circumstances. Since there were no Miranda warnings given, the confession is inadmissible. (Note that the lack of Miranda warnings would not necessarily preclude admission of the gun.) Answer choice A is incorrect because it would allow for admission of evidence derived from such a confession. Answer choices C and D are incorrect because they would allow for admission of an illegally obtained confession.

A manager runs a soup kitchen for malnourished children whose parents cannot afford to provide their children with freshly cooked, nutritious meals. So that the soup kitchen can remain financially viable, the manager often accepts free ingredients from a local food bank. One day, the manager received 400 pounds of potatoes from the food bank. He had very few other ingredients on hand that day, so he decided to serve a healthy potato soup for the children. When he started opening the potato bags, he noticed that a majority of the potatoes had green eyes and greenish skin. When he gave the potatoes to his volunteer chef, the chef mentioned that green potatoes were unsafe and potentially toxic, especially when fed to children. The manager brushed her off, and said that the green potatoes were just unripe. The volunteer chef refused to cook with the green potatoes, so she left and the manager made the potato soup by himself. A few weeks later, many of the children who ingested the potato soup ended up in comas. The doctors were able to trace the cause of the comas back to the toxic green potatoes. The doctors saved all but two of the children, who never came out of the comas and eventually died. What is the most serious offense of which the manager can be properly convicted?

Involuntary manslaughter. Involuntary manslaughter is an unintentional homicide committed with criminal negligence or during an unlawful act. Criminal negligence is grossly negligent action that puts another person at a significant risk of serious bodily injury or death. It requires more than ordinary negligence for tort liability and something less than the extremely negligent conduct required for depraved-heart murder. Here, it was grossly negligent for the manager to cook with the green potatoes, especially when the volunteer chef warned him that the green potatoes were potentially toxic when fed to children. Answer choice A is incorrect because common-law murder is the unlawful killing with malice aforethought, which includes a reckless indifference to an unjustifiably high risk to human life (depraved heart). There is a split among jurisdictions as to whether the requisite depravity exists when a defendant is actually unaware of the risk involved in the conduct, but the majority of states and the MPC impose liability only when the defendant actually realizes the danger. Here, the manager's conduct did not rise to the level of reckless indifference because although he was grossly negligent by cooking the green potatoes, he believed that they were just unripe. Thus, he lacked the malicious intent necessary to be found guilty of murder. Answer choice B incorrect because voluntary manslaughter involves an intentional killing committed under mitigating circumstances. Here, there was no intent to kill. Answer choice D is incorrect because the manager's conduct was greater than mere civil negligence.

A gardener entered a garden center through a side entrance during regular business hours. He intended to steal some heirloom parsley plants for his garden. He noticed a sales display of regular parsley plants at the back of the store, but mistakenly believed they were a very valuable type of heirloom parsley. When no one was looking, he took two small parsley plants from the sales display, put them under his sweatshirt, and left the store. At the register, there was a sign that advertised, "Like parsley? We like it so much that we grew more than we need this season! Ask about our overstock! Today only, we are offering each customer two free parsley plants from the surplus in our greenhouse!" The gardener never saw the sign. Burglary in the jurisdiction is defined as "entering any building unlawfully with the intent to commit a crime." The jurisdiction follows the common-law definition of larceny. The following crimes are listed in descending order of seriousness. Which is the most serious crime of which the gardener may be convicted?

Larceny only. The gardener is guilty of larceny because he took and carried away the property of the garden center with the intent to steal. Mistake of fact is only a defense if it negates the defendant's state of mind. Therefore, the gardener's mistaken belief that he was stealing a more valuable kind of heirloom parsley is not a defense because it did not negate his intent to steal. Additionally, even though surplus parsley plants from the greenhouse were being given away for free, the plants on display in the store were not. Therefore, the gardener has committed a larceny regardless of whether he saw the sign because he was unlawfully taking the property of the garden center with the intent to permanently deprive the store of the parsley plants. He is not guilty of burglary because there was no unlawful entering as required under the statute. The gardener entered the garden center lawfully as an invitee at a time when the garden center was open to the public. Thus, answer choices A and B are incorrect. Answer choice D is incorrect because, although the gardener did commit the crime of attempted larceny by taking a substantial step towards the completion of the crime of larceny, the crime of attempted larceny is a less serious crime than the completed crime of larceny.

A girl and her boyfriend went out for a night of eating and drinking. As usual, the boyfriend drank too much, and he ended up spilling red wine all over the girl's jacket. The jacket, which, because of its condition, was of little value, was treasured by the girl because it had been given to her by her great-grandmother. Upon returning to his apartment, the boyfriend promptly passed out in his bedroom. Angry at her boyfriend and intending to teach him a lesson, the girl took his valuable watch, which had been given to him by his grandfather, off of his wrist and put it in her purse. She planned to pawn it the first chance she got and keep the money. The girl went to bed next to her boyfriend, but in the morning she, feeling guilty, put the watch back on his nightstand before he woke up. What crime, if any, has the girl committed?

Larceny, because she took her boyfriend's watch with the intent to permanently deprive him of it.Larceny is the trespassory taking and carrying away of the personal property of another (without his consent) with the specific intent to permanently deprive the owner of the property at the time of the taking. The girl committed larceny. She took the watch off of her boyfriend's arm without his consent, she moved it at least a few steps from its original position, and she put it in her purse with the intent to permanently deprive him of it at the time of the taking. Answer choice A is incorrect because the girl completed the crime once she exerted dominion over the watch and carried it away, even a few steps. The boyfriend's awareness of whether a crime was committed is irrelevant. Answer choice B is incorrect because, although her purpose may have been "to teach her boyfriend a lesson," the girl still had the requisite intent to permanently deprive him of the watch when she took it. Answer choice C is incorrect because, although larceny does require a taking and carrying away of personal property, here the girl removed the watch from her boyfriend's wrist and placed it in her own purse. This is sufficient to satisfy the asportation requirement for larceny.

A man was married to his childhood sweetheart, but after twenty years of marriage, they were no longer in love, and the man took on a mistress. One day, the man's wife disappeared. The next day, believing his wife to be dead, the man married his mistress. However, the wife was not dead, but had merely been visiting her sister. When she returned home a week later, she found her husband married to the mistress. The man told his wife that because their religion required polygamy, being married to two women was his religious duty. The wife disagreed and subsequently reported her husband to the police. Will the man likely be convicted of bigamy?

Mens rea is the requirement of a guilty mind or legally proscribed mental state that a defendant must possess to commit a crime. Except for strict liability crimes, a crime is committed when a criminal act (actus reus) is coupled with a guilty mind—both the mental and physical elements exist at the same time. Strict liability crimes have no mens rea requirement and require only an actus reus. Bigamy is a strict liability crime, thus only an actus reus is required. An actus reus is a criminal act that must be a voluntary, affirmative act that causes a criminally proscribed result. In this case, even if the man thought his wife was dead and believed his marriage to the mistress to be legal, he will likely be found guilty of bigamy because he voluntarily married the mistress while still married to his wife. Answer choice A is incorrect because the wife's objection is immaterial to a bigamy charge. Even if she consented to the marriage, the man's actions are still illegal. Answer choice C is incorrect because it is only relevant that the man intended to marry the mistress. Answer choice D is incorrect because religious belief is not a defense to the crime of bigamy.

The owner of a parcel of land constructed a house on the land and created a paved driveway from the house to a public road that ran along the western boundary of his property. Shortly thereafter, the adjacent parcel of land to the north of the owner's parcel was sold. Although this parcel had direct access to the public road, the buyer entered into an agreement to purchase an easement from his neighbor, the owner, for a small fee. This agreement was promptly and properly recorded. Under its terms, the buyer obtained the right to use the owner's driveway to access the road from his property. Twenty years later, the owner's daughter holds title to the owner's parcel, which she inherited from the owner. Last year, the buyer sold his parcel to a friend, who is the owner of the adjacent parcel of undeveloped land to the buyer's north. This third parcel has direct access to the public road. The friend plans to tear down the house constructed by the buyer and build new home on the third parcel of land. The friend told the daughter of his plans and mentioned that he planned to continue using her driveway to access his new residence. She objected and has told him that she will construct a barrier on her land to prevent him from doing so, even though the burden on her property from the use of her driveway would not be increased. If the friend brings suit seeking to prevent the daughter from blocking access to her driveway, who should prevail?

The daughter, because the easement was appurtenant. Easements are presumed to be appurtenant (i.e., tied to the land) unless there are clear facts to the contrary. The benefits of an easement must correspond directly to the use and enjoyment of the possessor of the dominant estate. Here, the easement granted by the owner to the buyer of the adjoining parcel was appurtenant rather than in gross. It was granted to the buyer as the owner of the adjoining parcel for the buyer to use the owner's driveway to access the public road from his property. Consequently, although the easement could be transferred by the buyer to his friend, it is confined to the use of the buyer's parcel. The friend is attempting to expand the dominant estate to include a third parcel that was not owned by the buyer at the time that the easement was created. The friend cannot compel the daughter to accept this expansion of the dominant estate to which the easement is subject. Answer choice A is incorrect. A change in the scope of an existing easement is tested under a reasonableness standard in light of the purpose of the easement, when the change does not conflict with the terms of the easement. Here, although the friend is proposing to use the easement for the same purpose, access to one house, and that use will not increase the burden on the servient estate, the easement was granted for the use of the parcel owned by the buyer. The dominant estate did not include the adjoining third parcel owned by the friend. Consequently, the friend cannot expand the dominant estate to include that parcel. Answer choice B is incorrect because the recording of the easement has no effect on the ability of the friend to enforce the easement against the daughter. As she took ownership of the servient estate by devise rather than purchase, she would not be protected by the recording act. Answer choice C is incorrect. Although an easement can be terminated by the destruction of the related structure, this typically involves the destruction by natural forces of a structure on the servient estate to which the easement provided access. Here, the destruction of the house on the dominant estate by the friend does not terminate the reason for the easement (i.e., providing access to the public road).

A plaintiff brought a state law action for medical malpractice against a defendant in federal district court under diversity jurisdiction. The plaintiff's complaint alleged only that the defendant's negligent conduct caused the plaintiff's damages. The complaint did not include any specific factual allegations regarding the defendant's conduct. The defendant has not yet filed an answer to the complaint. In response to the complaint, which of the following motions would be most appropriate for the defendant to file?

Motion to dismiss for failure to state a claim upon which relief can be granted. A complaint should not be dismissed for an mere imperfect statement of the legal theory. However, under Rule 12(b)(6), a claim for relief can be dismissed if it either fails to assert a legal theory of recovery that is cognizable at law or fails to allege facts sufficient to support a cognizable claim. In reviewing a motion to dismiss under Rule 12(b)(6), the court must identify and reject legal conclusions unsupported by factual allegations. This includes mere conclusory statements and assertions devoid of facts. In this case, the sole allegation in the plaintiff's complaint was a mere conclusory statement. The plaintiff failed to include any factual allegations to support the legal conclusion that the defendant's negligent conduct caused the plaintiff damages. Thus, the complaint should be dismissed for failure to state a claim upon which relief can be granted. Answer choice A is incorrect. A motion to strike is appropriate if a pleading contains any insufficient defense, or redundant, immaterial, impertinent, or scandalous material. None of these are present here. Answer choice B is incorrect. A motion for summary judgment occurs after discovery is complete. Here, the defendant has not yet filed an answer to the complaint, and so it is too early in the litigation to bring a motion for summary judgment. Answer choice C is incorrect. A motion for judgment on the pleadings under Rule 12(c) must be made after an answer is filed. Here, the defendant has not filed an answer to the complaint.

An employee was up for a promotion but was passed over by his boss for a female colleague with more experience. After learning he had not received the promotion, he became angry with his boss and convinced himself that the colleague and the boss, who was married, were involved in a relationship. He therefore contacted the boss's wife and convinced her to shoot the boss. They stated that they would not harm the colleague, as it might make their involvement too obvious. The employee provided her with a gun. The next day, the boss's wife approached the colleague and the boss in their office parking lot. The wife, who was not an experienced shooter, shot the colleague in the arm, and then panicked and ran off. An onlooker rushed the colleague to the hospital. Although the injury was not life threatening, she contracted an infection during surgery and died the following week. A later investigation revealed that the infection was a result of medical malpractice that occurred during the surgery. The crime was eventually traced back to the employee and the boss's wife, and they were both charged in connection with the colleague's death. The employee is most likely to be convicted of which of the following crimes?

Murder, attempted murder, and conspiracy to commit murder. To prove a homicide, the prosecution must show that the defendant was the actual and proximate cause of the victim's death. If the victim would not have died but for the defendant's act, then the defendant's act is the actual cause of the killing. To prove proximate cause, the death must be foreseeable. A defendant's conduct is deemed to be foreseeable if death is the natural and probable result of the conduct. Actions by a third party (e.g., negligence by the doctor treating the victim) are generally foreseeable. In this case, the boss's wife was the actual and proximate cause of the colleague's death, and thus would be guilty of murder of the colleague. Although the crimes of murder and attempted murder would usually merge, they only merge as to the same person. While the boss's wife could be found guilty of the colleague's murder, she could also be found guilty of the separate crime of attempting to kill the boss. The wife had the specific intent to kill the boss, and took a substantial step towards that result (approaching the boss in the parking lot and shooting the gun). The employee would be guilty as a co-conspirator. Conspiracy is an agreement between two or more persons to accomplish an unlawful purpose with the intent to accomplish that purpose. A conspirator can be convicted of both the offense of conspiracy and all substantive crimes committed by any other co-conspirators acting in furtherance of the conspiracy. In this case, the employee and the wife agreed to kill the boss, and the employee committed an overt act—providing a gun to the wife—in furtherance of the conspiracy. Thus, the employee would be guilty of both murder and conspiracy, even though the person killed was actually the colleague and not the boss. Answer choice B is incorrect because it fails to take into account the attempted murder of the boss. Answer choice C is incorrect because the employee would be guilty of murder, because the colleague's death was a natural and probable cause of the wife's actions. Answer choice D is incorrect because the employee would also be guilty of the attempted murder of the boss; because the boss was not actually murdered, that attempt would not merge with the actual conviction for the colleague's murder.

While climbing a steep mountain, a woman's climbing rope failed, and she fell off the side of the mountain. She landed on a small ledge 30 feet below. Behind her, a man continued climbing. The woman shouted, "Hey! I'm here! Throw me a rope so I can get up!" The man, who did not know the woman but had all the gear that would be necessary to safely rescue her, looked down at her and said, "Sorry, I'd like to help, but I have to reach the summit before dark." He continued on to the summit. On the way back down, he looked down to see that the woman had fallen off the ledge to her death. What is the most serious crime for which the man can be convicted?

No crime. A legal duty to act and the failure to do so results in criminal liability in certain instances, such as when there is a special relationship between the parties (e.g., a parent's duty to her child) or when a party fails to aid the victim after causing the victim's peril. When there is no duty to act, a defendant is not criminally liable just because he fails to help others in trouble. In this case, the woman and the man do not have a special relationship, as it appears they are strangers. Neither did the man cause the woman to fall down the side of the mountain onto the small ledge or voluntarily assume to render aid. Thus, the man had no legal duty to act. Although the woman died and the man could have prevented her death, he is not liable for her death. Answer choices A, B, and C are therefore incorrect.

A woman was out late drinking at a party, and decided to walk home rather than drive home drunk. However, she was very inebriated, and lost the keys to her house. It was very cold outside, so she decided to sneak into her neighbor's home for a few hours to stay warm until morning. He happened to be out of town, but she knew that one of the ground floor windows could be pried open easily and there was no alarm system. She entered their home through the loose window, covered up under some warm blankets, and slept for a few hours. When she woke up, she had sobered up, and as she was leaving, she grabbed her neighbor's heavy coat because it was still cold outside, planning to return it next time she saw him. The jurisdiction in question only recognizes trespassing as a tort, not as a criminal offense. What is the most serious crime, listed below in ascending order of seriousness, of which the woman can be convicted?

No crime. The woman lacked the requisite intent to commit either of the crimes listed, as explained below. Accordingly, she would not be found guilty of either crime. Answer choice B is incorrect because although necessity can be used as a defense if forces of nature cause the defendant to commit what would otherwise be a crime, there was no crime in this case. Therefore, the defense of necessity is not applicable. Even if the woman's trying to escape the cold after losing her keys while inebriated constituted necessity, which is unlikely, the woman would not need to raise a defense because she did not complete the elements of either crime. Answer choice C is incorrect because larceny is the trespassory taking and carrying away of the personal property of another with the intent to permanently deprive him of it. The woman attempted only to borrow the coat, with the intent to return it, so she cannot be convicted of larceny. Answer choice D is incorrect because to be guilty of burglary, at the time of the breaking and entering, the defendant must have the intent to commit a felony (e.g., larceny, robbery, rape, murder) inside the dwelling. The defendant did not have the intent to commit larceny (or any other crime) when she entered the neighbor's home; she just wanted to stay warm.

A lumberyard contracted with a retail home improvement company to provide the company with pine boards of various lengths and amounts. The contract was silent as to delegation, and the lumberyard, acting in good faith, delegated its duties under the contract to a third party. After asking for, and receiving, assurances from the third party that the boards would conform to contract specifications, the company accepted the delegation. The third party delivered the boards, which the company rejected because the shipment did not conform to its specifications. The company sued both the lumberyard and the third party. The lumberyard moved to dismiss the claims against it. Should the court dismiss the suit against the lumberyard?

No, because a delegation of duties does not absolve the lumberyard of its contractual liability.Answer choice D is correct. When obligations are delegated, the delegator is not released from liability, and recovery can be had against the delegator if the delegate does not perform, unless the other party to the contract agrees to release that party and substitute a new one (a novation). Consequently, the lumberyard remained liable on the contract. Answer choice A is incorrect because, while a party to a contract may be released from liability if the other party agrees to release that party and substitute a new one (a novation), mere acceptance of a delegation does not constitute a novation, and the lumberyard remained liable on the contract. Answer choice B is incorrect because the mere fact that a party to a contract delegates its duties in good faith does not eliminate its duties to perform under the contract. Answer choice C is incorrect because it misstates the law. There is no UCC provision that prevents the delegation of duties unless the contract specifically provides for it.

An indigent defendant was tried for robbery. She was found guilty. As permitted by state law as a matter of right, the defendant sought to appeal her conviction. Her state-appointed attorney timely filed a notice of appeal. The defendant instructed her attorney to raise a specific claim of error on appeal. The attorney, acknowledging that the claim was a nonfrivolous, colorable claim, decided for tactical reasons to appeal on other grounds, which the attorney felt, based on his reasonable professional judgment, had more merit. The appellate court upheld the defendant's conviction. The defendant subsequently challenged the decision of the appellate court on the grounds of ineffective assistance of counsel under the Due Process Clause of the Fourteenth Amendment. Will the defendant's challenge likely be successful?

No, because an appointed attorney's exercise of reasonable professional judgment in determining which issues to advocate on behalf of a client on appeal does not constitute ineffective assistance of counsel. The first part of the two-part test for establishing ineffective assistance of counsel requires that the representation of a defendant by the defendant's attorney fall below an objective standard of reasonableness. While an appointed attorney is generally required to pursue a nonfrivolous appeal on behalf of an indigent defendant, in pursuing such an appeal, the attorney is not required to advance every nonfrivolous claim of error urged by the defendant, but may use his professional judgment in determining which claims of error to assert. So long as this judgment is reasonable, the defense attorney's conduct does not form a basis for ineffective assistance of counsel. Answer choice A is incorrect because, as explained above, an attorney may use his professional judgment in determining which claims to assert, and need not advance all nonfrivolous claims. Answer choice B is incorrect for the same reason. Answer choice D is incorrect because, where the state grants an appeal as of right, an indigent defendant must be provided with an attorney.

A mother was frustrated with her four-year-old child, who would not stop throwing a tantrum in a large chain department store. After screaming that he wanted a toy over and over again, and trying to run away from her to the toy aisle, the mother swatted her son on the behind. Another woman in the store saw the mother swat the son and saw the child cry and grab his behind; because she believed that spanking was child abuse, she reported the mother to the police. The police arrested the mother for battery. When they examined the child, he had no marks on his body. Will the mother likely be convicted of battery?

No, because as a parent she is permitted to discipline her child. The use of reasonable force in the exercise of parental authority (i.e., discipline) by a parent or by a person in charge of a child is justified if exercised for the benefit of the minor child. Here, the mother swatted her child to improve his behavior, and she merely swatted him; she did not physically harm him. Therefore, the mother's actions were within her right to discipline her child. Answer choice A is incorrect. Although the force that the mother applied to her son when swatting him on the behind would support a finding of battery, the mother was justified in doing so as a proper exercise of parental authority. Answer choice B is incorrect because there is no limitation that parental discipline be delivered only in private. Answer choice C is incorrect because it is incomplete. It is true that the mother did not physically harm her son. However, a battery must be a touching to the person of another. The touching, however slight, must result in bodily harm (e.g., a bruise) or an offensive touching (e.g., an unwanted kiss). Here, the swat was likely offensive to the son. Thus, the mother would be convicted for battery except for the fact that she had a defense as a parent of a minor child.

An insurer, having paid the claim of the owner of a building that was destroyed by fire, properly filed an action in federal district court sitting in diversity against a tenant in the building. The insurer, asserting that it was subrogated to the rights of the owner of the building, sued the tenant to recoup the amount the insurer had paid to the building owner. The insurer asserted that the tenant's negligence caused the fire. A second tenant timely filed a motion to intervene in this action, asserting a claim against the first tenant for its own losses that it suffered as a consequence of the fire. The second tenant's losses exceeded $100,000. Both tenants are citizens of the same state. Does the court have subject matter jurisdiction over the second tenant's claim against the first tenant?

No, because both tenants are citizens of the same state. In order for a federal court to have subject matter jurisdiction over a claim based on diversity jurisdiction, both the amount-in-controversy and the diversity-of-citizenship requirements must be met. Here, although the second tenant's claim satisfies the amount-in-controversy requirement, it does not satisfy the diversity-of-citizenship requirement because the second tenant and the first tenant are both citizens of the same state. Similarly, the court does not have supplemental jurisdiction over this claim because the exercise of jurisdiction over the claim would defeat the diversity-of-citizenship requirement. Supplemental jurisdiction does not apply to the claims of a person seeking to intervene in a case based exclusively on diversity jurisdiction if the exercise of jurisdiction would be inconsistent with the requirements of diversity jurisdiction. Consequently, the court lacks subject matter jurisdiction over this claim. Answer choice B is incorrect. It is likely that the second tenant's intervention does not constitute an intervention as of right, but regardless of whether the second tenant's intervention is as of right or permissive, the court lacks subject matter jurisdiction over the second tenant's claim. Answer choice C is incorrect. Although the second tenant's claim satisfies the amount-in-controversy requirement, it does not satisfy the diversity-of-citizenship requirement. Answer choice D is incorrect because supplemental jurisdiction does not apply to the claims of a person seeking to intervene under Rule 24 (either as of right or permissively) in a case based exclusively on diversity jurisdiction if the exercise of jurisdiction would be inconsistent with the requirements of diversity jurisdiction.

An artist filed a negligence suit against a teacher. The artist was injured when the teacher's car crashed into her motorcycle. At trial, the artist called the teacher's ex-husband as a witness during her case-in-chief to testify regarding the teacher's driving record, including two previous civil suits against the teacher for reckless driving that resulted in verdicts against the teacher. These previous lawsuits occurred after the teacher and her ex-husband had already divorced. The artist contends that the ex-husband's testimony tends to show that the teacher is prone to reckless driving. The teacher objects to the artist calling her ex-husband to testify for this purpose. Is this testimony admissible?

No, because it cannot be used to prove the teacher drove recklessly in this case.In a civil case, evidence of an individual's character (or character trait) is generally not admissible to prove that the person acted in accordance with that character (or character trait) on a specific occasion. In this case, the artist is offering the ex-husband's testimony about the prior lawsuits to prove that the teacher drove recklessly in this case. This is an impermissible use of character evidence, and thus the testimony is not admissible. Answer choice A is incorrect. The prior verdicts against the teacher are not admissible to prove her recklessness in this case, even if they are a matter of public record. Answer choice B is incorrect. Regardless of whether the ex-husband's testimony is relevant, it is not admissible to prove the teacher drove recklessly in this case. Answer choice C is incorrect. The confidential marital communications privilege protects communications made between spouses while they were married if those communications were made in reliance on the sanctity of marriage. Here, the previous lawsuits against the artist occurred after she and her ex-husband were divorced, so any confidential communications about the lawsuits between the teacher and her ex-husband would not be privileged.

An older man was corresponding daily with a 13-year-old girl in an online chat room called Minors for Men. The older man knew that the girl was a minor because they discussed her age and the trials and tribulations of being a teenager during their many online chats. In addition, the girl sent a picture of herself to the older man and it was clear that she was a minor. The two agreed to meet, and they ended up having sexual intercourse. The older man's wife discovered the relationship when she was looking through his Internet history. She reported him immediately to the authorities and the older man was arrested. The girl found out, and revealed to the authorities that she was actually 23 years old, but she looked very young and she enjoyed role-playing. Can the older man be found guilty of attempt to commit statutory rape?

No, because it was legally impossible for the older man to commit statutory rape of a 23-year-old woman.Statutory rape is sexual intercourse with a person under the age of consent. The man intended to have sexual intercourse with a minor. Impossibility is not a defense to attempt if the crime attempted is factually impossible to commit due to circumstances unknown to the defendant. If the girl had actually been only 13 years old, then there would have been a crime committed. Further, the MPC crime of attempt (as well as many jurisdictions) requires the performance of a substantial step to support a conviction of attempt, which the man performed here. Answer choice A is therefore incorrect. Answer choice B is incorrect because legal impossibility does not apply to this situation. Legal impossibility is a defense if the act intended is not a crime; the defendant cannot then be guilty of attempt. Statutory rape is a crime, so the older man is guilty of attempt. Answer choice D is incorrect because statutory rape is a strict-liability crime with respect to the age of the victim. Thus, a defendant's reasonable mistake of fact concerning the victim's age is not a defense.

A man and a woman agreed to engage in consensual sexual intercourse at a particular hotel the following night. The man, who was the son of the woman's brother-in-law, rented the hotel room for their tryst, but the woman did not show up. In the applicable jurisdiction, a statute provides that a person commits the offense of incest when such person engages in sexual intercourse with a person whom he or she knows he or she is related to either by blood or by marriage as aunt or nephew. Can the woman and man properly be charged with conspiracy to commit incest?

No, because of the Wharton Rule. Conspiracy is (i) an agreement (ii) between two or more persons (iii) to accomplish an unlawful purpose (iv) with the intent to accomplish that purpose. Under the majority rule, one of the co-conspirators must also commit an overt act in furtherance of the conspiracy. Here, a woman and man who were related to each other as aunt and nephew by marriage agreed to engage in consensual sexual intercourse and the man rented a hotel room for that purpose. However, under the Wharton Rule, if a crime requires two or more participants, there is no conspiracy unless more parties than are necessary to complete the crime agree to commit the crime. In this case, because only the woman and man agreed to commit incest and both are required in order to commit this crime, neither can properly be charged with the crime of conspiracy to commit incest. Answer choice A is incorrect. Although each agreed to commit incest, because both are necessary parties to this crime, neither can be charged with conspiracy to commit incest. Answer choice B is incorrect. The fact that the man rented a hotel room would constitute an overt act in furtherance of the conspiracy. However, as both the woman and the man are necessary parties to this crime, neither can be charged with conspiracy to commit incest. Answer choice D is incorrect because, upon completion of the overt act, the conspiracy is formed, and withdrawal is no longer possible. Consequently, the woman's failure to show up would not serve as a withdrawal from the conspiracy.

Pursuant to an act of Congress, a new executive department was created following a rise in the number of threats to security in the country. The new executive department was headed by a commission comprised of five officials selected by the President with Senate approval. The act authorizing the creation of the new executive department also mandated the creation of a number of lower-level committees charged with investigating security threats. The committees, which would be supervised by a Senate-confirmed appointee, would then make reports to the commission regarding their investigations. Congress permitted the President to appoint the members of the investigatory committees. The President appointed the members of the committees without seeking Senate approval.

No, because the President needs the consent of the Senate to make these appointments. Congress may delegate the appointment of "inferior" officials to the President. "Inferior" officials are those supervised by Senate-confirmed appointees. Here, Congress can delegate to the President the power to appoint these inferior officials without Senate approval because they will be supervised by a Senate-confirmed appointee. Answer choice A is incorrect because, as stated above, Congress can delegate the appointment of inferior officials to the President. Answer choice B is incorrect because the President does not need Senate approval to make these appointments. Answer choice C is incorrect. Congress may not itself appoint members of a body with administrative or enforcement powers. Such persons are "officers of the United States" and must be appointed by the President with the advice and consent of the Senate. Congress may, however, delegate the appointment of "inferior" officials to the President alone (i.e., without Senate approval), the heads of executive departments, or the courts.

A collector of historical documents discovered that a letter she had purchased many years earlier that she thought was in the handwriting of a famous historical figure was a forgery. Because the person who sold her the letter was now dead, the collector advertised the letter for sale as genuine and priced it accordingly. The benefactor of a museum expressed interest in purchasing the letter. Before doing so, the benefactor had a curator at the museum examine the letter. The curator mistakenly informed the benefactor that the letter was genuine. As a consequence, the benefactor purchased the letter, paying the collector in cash. Can the collector be charged with false pretenses?

No, because the benefactor did not rely on the collector's representation of the letter as genuine. False pretenses requires (i) obtaining title to the property (ii) of another person (iii) through the reliance of that person (iv) on a known false representation of a material past or present fact, and (v) the representation is made with the intent to defraud. Here, although the collector advertised the letter as genuine and obtained money from the benefactor for the letter, the benefactor did not purchase the letter in reliance on the collector's representation, but instead on the museum curator's assessment of the letter's authenticity. Consequently, the collector cannot properly be charged with false pretenses. Answer choice A is incorrect. Although the collector did not forge the letter, the collector did misrepresent its authenticity. Consequently, had the benefactor purchased the letter in reliance on the collector's misrepresentation, the collector could have been charged with false pretenses even though the collector did not forge the letter. Answer choice C is incorrect because, although the collector misrepresented the authenticity of the letter, the benefactor did not rely on this misrepresentation in purchasing the letter. Answer choice D is incorrect. Although the collector did receive money from the benefactor for the letter and thus obtained title to the property of another, the benefactor not did rely on the collector's misrepresentation, and therefore the collector cannot properly be charged with false pretenses.

The owner of a home was awakened in the middle of the night by the sound of breaking glass. Erroneously concluding that he had been dreaming, the owner went back to sleep. Several minutes later, the owner was again awakened by a loud crashing sound coming from his living room. The owner grabbed a pistol that he kept by his bedside and went to investigate. In his living room, the owner saw a burglar place the owner's laptop computer into a bag and then watched as the burglar quickly headed for the back door of the house. Before the burglar could escape, the owner intentionally shot and killed the burglar. The owner has been charged with the murder of the burglar. Is the owner's killing of the burglar justified?

No, because the burglar was heading for the back door. Although generally there is no right to use deadly force in defense of property, a person may use deadly force to prevent or terminate forcible entry into a dwelling if the occupant reasonably believes that the intruder intends to commit a felony inside. However, the use of deadly force against an intruder exiting the dwelling is generally not permissible. Here, although the owner had the right to use deadly force against the burglar in order to prevent the burglar from stealing property from the owner's home, the owner did not have that right to prevent the burglar from exiting the home. Therefore, the owner's killing of the burglar was not justified. Answer choice A is incorrect. Although a property owner generally cannot use deadly force to defend his property, a person may use deadly force to prevent or terminate forcible entry into a dwelling if the occupant reasonably believes that the intruder intends to commit a felony inside. Answer choice C is incorrect. Although the owner had the right to use deadly force against the burglar in order to prevent the burglar from stealing property from the owner's home, the owner did not have the right to prevent the burglar from exiting the home. Thus, the fact that the victim was a burglar does not justify the owner's killing. Answer choice D is incorrect. Although a homeowner may use self-defense to protect himself or others from the threat of deadly force and is not required to retreat when in his own home before doing so, the owner here was not acting in self-defense or in defense of others, but in defense of his property.

A new Food and Drug Administration ("FDA") regulation mandated that within one year after the effective date of the regulation, manufacturers of children's cereals must remove from their boxes or advertisements any words or images that refer to animated characters from children's movies and television shows, or be subject to severe penalties. In response to the increasing problem of childhood obesity, the FDA promulgated the regulation to help children learn to make food choices that are not based solely upon their favorite animated character. One year after the regulation was promulgated, a cereal manufacturer, who did not manufacture children's cereal but worried about further meddling by the FDA into cereal advertising, filed suit in federal court to enjoin the regulation as an unconstitutional restriction on its commercial speech rights. Can a federal court determine the merits of this suit?

No, because the cereal manufacturer cannot show an injury in fact to establish standing. To have standing, a plaintiff bears the burden of establishing three elements: (i) there is an injury in fact, (ii) the injury must be caused by the defendant's violation of a constitutional or other federal right, and (iii) the relief requested must prevent or redress the injury. While the threat of future injury can suffice, it cannot be merely hypothetical or conjectural, but must be actual and imminent. Here, the cereal manufacturer has not suffered a direct injury, and because it does not manufacture children's cereal, any threat of a future injury is purely hypothetical. Therefore, the cereal manufacturer lacks standing, and the federal court cannot determine the merits of the suit. Answer choice A is incorrect. Even though the regulation is now enforceable, the cereal manufacturer has suffered no injury and cannot establish an actual imminent threat that the regulation will be enforced against it. Answer choice B is incorrect. Although the federal government can hear a suit for an injunction against a federal regulation, the suit must still meet the "case or controversy" requirement in order to be heard, and the plaintiff must have proper standing. Answer choice D is incorrect because it is overly strict; an imminent threat of injury may be sufficient to create standing and ripeness.

A defendant agreed to help a friend with a burglary by transporting the friend to the scene of the burglary in the defendant's car, keeping watch outside the residence while the friend committed the burglary, and then driving the friend away from the scene. The friend believed that no one would be home at the time of the burglary, so the friend told the defendant he would be unarmed. He lied, and instead carried a gun on his person into the residence. After waiting for several minutes, the defendant got cold feet and drove away from the residence, abandoning his friend. Subsequently, the friend, still inside the residence, encountered a family member. Panicking, the friend shot and killed the family member. The defendant has been charged with felony murder in a state that permits capital punishment by lethal injection for felony murder. Can the death penalty be imposed on the defendant?

No, because the defendant did not kill, attempt to kill, or intend to kill the family member. In felony-murder cases, the death penalty may not be imposed if the defendant, acting as an accomplice, did not kill, attempt to kill, or intend to kill, unless the defendant significantly participated in the commission of the felony and acted with reckless indifference to human life. Answer choice A is incorrect. Although the death penalty may be imposed on a defendant who is convicted of felony murder, it may not be imposed on an accomplice-defendant, such as the defendant in this case, who does not kill, attempt to kill, or intend to kill the victim. Answer choice B is incorrect. Death by lethal injection is not generally considered cruel and unusual punishment because there is only a mere possibility that the condemned may receive an improperly administered shot that would cause him unnecessary pain. However, subjecting the defendant in this case to the death penalty is not permissible under the Eighth Amendment because he did not kill, attempt to kill, or intend to kill. Answer choice D is incorrect because there are situations when the death penalty may be imposed for a defendant convicted of felony murder.In felony-murder cases, the death penalty may not be imposed if the defendant, acting as an accomplice, did not kill, attempt to kill, or intend to kill, unless the defendant significantly participated in the commission of the felony and acted with reckless indifference to human life. Answer choice A is incorrect. Although the death penalty may be imposed on a defendant who is convicted of felony murder, it may not be imposed on an accomplice-defendant, such as the defendant in this case, who does not kill, attempt to kill, or intend to kill the victim. Answer choice B is incorrect. Death by lethal injection is not generally considered cruel and unusual punishment because there is only a mere possibility that the condemned may receive an improperly administered shot that would cause him unnecessary pain. However, subjecting the defendant in this case to the death penalty is not permissible under the Eighth Amendment because he did not kill, attempt to kill, or intend to kill. Answer choice D is incorrect because there are situations when the death penalty may be imposed for a defendant convicted of felony murder.

During discovery in a state court action, an attorney inadvertently disclosed an impromptu letter sent by his client to the attorney regarding the client's thoughts and feelings about the litigation. Belatedly, the attorney took steps to rectify the error. Under the applicable state law, the disclosure did not constitute a waiver of the attorney-client privilege because it was inadvertent. In subsequent litigation in a federal action to which the client was a party, the opposing party sought discovery of the letter, which was relevant to that party's claim. Does the attorney's prior inadvertent disclosure of the letter constitute a waiver of the attorney-client privilege for purposes of the federal action?

No, because the disclosure did not constitute a waiver under state law. Under the federal rule, an attorney's inadvertent disclosure of a communication that is protected by the attorney-client privilege can operate as a waiver of the privilege when there is a failure by the attorney to promptly take reasonable steps to rectify the error. However, this rule does not apply when the disclosure was made in a state court proceeding and the applicable state law does not treat the disclosure as a waiver. When state law is more protective of the attorney-client privilege than the federal rule, applicable state law governs and the disclosure is not treated as a waiver in a subsequent federal proceeding. Answer choice B is incorrect because although a letter detailing a client's mental impressions is protected by the attorney-client privilege, the privilege may be waived in certain circumstances. Answer choice C is incorrect because, while the disclosure would constitute a waiver of the attorney-client privilege under the federal rule on inadvertent disclosures, the state law applies to determine the effect of the waiver. Answer choice D is incorrect because the applicable state law rather than the federal rule determines the effect of the disclosure in this case.

An environmentalist wanted to expose the amount of pollution being created by a power plant that used coal-burning power generators. To do this, the environmentalist broke into the power plant in order to steal the plant's monthly reports regarding the amount of carbon dioxide it released. The environmentalist successfully stole the reports. He then anonymously sent the report to the editor of a major environmental magazine, who used the stolen reports to write and publish a feature story about the amount of carbon dioxide pollution caused by the plant. If the plant files suit against the editor for the publication of illegally obtained private information, will it succeed?

No, because the editor played no part in unlawfully obtaining the monthly reports. The press has the right to publish information about matters of public concern, and the viewers have a right to receive it. The First Amendment shields the media from liability for publishing information that was obtained illegally by a third party as long as the information involves a matter of public concern and the publisher did not obtain it unlawfully. Here, the information regarding the amount of carbon dioxide pollution generated by the power plant is of great public concern. The environmentalist illegally obtained the monthly reports, but he sent them to the editor anonymously and not at the direction of the editor. Therefore, the editor is not liable for publishing the information. Answer choice A is incorrect because it is too broad. Although the press does have the right to publish information about matters of public concern, it cannot on its own illegally obtain information, and there may be governmental interests that warrant the prevention of publication. Answer choice C is incorrect because the press may have the right to publish private facts if they pertain to a matter of public concern. Answer choice D is incorrect because the First Amendment shields the media from liability for publishing information that was obtained illegally by a third party.

A defendant is on trial for a series of rapes. Based on victim accounts, the rapist would show up at the victims' homes with a bouquet of yellow roses blocking his face, and then spray the victims in the face with mace to prevent them from seeing his face. All of the victims were attacked after providing their personal information to the same profile on a dating site, a profile later linked to the defendant. After obtaining a valid search warrant for the defendant's home, the police searched the home but found no incriminating evidence. The police then searched the backyard, which abutted a public road and was not enclosed by a fence. They discovered a large plot of yellow roses along the road approximately 50 yards away from the home. Upon closer inspection, the police discovered that some of the rose bushes were adorned with personal trinkets reported as missing from each of the rapist's victims. Can the defendant successfully move to exclude evidence of the trinkets?

No, because the incriminating character of the yellow roses was immediately apparent, and the officers were in the back yard for a lawful purpose.Only unreasonable searches and seizures are subject to the Fourth Amendment. A search occurs when governmental conduct violates a reasonable expectation of privacy. Here, the rose garden was in the backyard along a public road and approximately 50 yards from the home. The backyard was not enclosed by a fence. There is no evidence that the defendant took steps to protect the rose garden adorned with trinkets from observation by the public, so it is not protected as a part of the curtilage of his home. Therefore, he had no reasonable expectation in the garden, and the trinkets need not be excluded. Answer choice A is incorrect because it states the standard for what may be seized in plain view in private areas, such as a home. Here, the rose garden is in an open field along a public road. Therefore, the incriminating character of the roses is irrelevant. Answer choice C is incorrect because the garden is not protected as the curtilage of the home. In determining whether an area is protected as curtilage, the following four-factor test applies: (i) the proximity of the area to the home; (ii) whether the area is included within an enclosure surrounding the home; (iii) the nature of the uses to which the area is put; and (iv) the steps taken by the resident to protect the area from observation by passersby. This garden was 50 yards from the home in an unenclosed field, and it was along a public road. Under these facts, the garden is not part of the curtilage of the home. Answer choice D is incorrect because the police do not need a warrant to seize this evidence. It was in plain view in a public place, so there was no reasonable expectation of privacy requiring a warrant.

A couple purchased a residence with the proceeds of a loan from a savings and loan association. The couple gave the savings and loan association a promissory note that was secured by a mortgage on the residence. When the outstanding balance on the couple's loan was $100,000 and the fair market value of the residence was $140,000, the couple sold the residence to an investor, who paid the couple $40,000 in cash. The investor did not assume the mortgage. The investor did not move into the residence, but instead rented it out to a third party. The loan, which contains an acceleration clause, is in default. May the savings and loan association pursue an action against the investor personally for the unpaid balance on the loan?

No, because the investor did not assume the couple's mortgage. Because the investor did not assume the couple's mortgage, the investor is not personally liable for the remaining balance due under the loan to the savings and loan association, even though the investor's ownership of the residence is subject to the mortgage. Answer choice B is incorrect because the investor's failure to use the premises as a residence is irrelevant to the investor's personal liability for the unpaid balance of the loan. Answer choice C is incorrect because, even though the loan contains an acceleration clause and is in default, which does give the savings and loan association the right to foreclose on its mortgage or seek the personal liability of the couple, the investor is not personally liable for repayment of the loan. Answer choice D is incorrect because, even though the investor did not pay the full fair market value of the property and the amount paid reflects a credit for the balance of the mortgage obligation, the investor is not personally liable unless the investor expressly assumes that obligation. Here, the facts clearly state that the investor did not do so.

In a negligence action that was brought in federal district court on the basis of diversity jurisdiction, the court submitted the case to the jury. The jury, after being properly instructed, was supplied with a verdict form that contained both specific interrogatories as well as a general verdict. In answering the interrogatories, the jury found that the plaintiff was 55 percent at fault and the defendant was 55 percent at fault. The jury's general verdict was in favor of the defendant. The applicable jurisdiction follows a modified comparative fault scheme under which a plaintiff is barred from recovering damages if the plaintiff's fault is greater than the defendant's. Based on the jury's general verdict, the court approved for entry by the clerk a judgment in favor of the defendant. Is the court's action proper?

No, because the jury's answers to the special interrogatories are inconsistent. When a jury is instructed to deliver both a general verdict and to answer special interrogatories, if the answers to the interrogatories are consistent with each other but not with the general verdict, the court has the option of approving a judgment that is consistent with the answers, notwithstanding the general verdict. However, when the jury's answers to those interrogatories conflict with each other, the court cannot enter a judgment and cannot approve the entry of the general verdict. In this instance, the jury, through its answers to the special interrogatories, found that the plaintiff and the defendant were each more at fault than the other (each was found to be 55 percent at fault). Consequently, the answers are inconsistent with each other and the court could not approve the entry of the general verdict. Instead, the court may either set aside the verdict and ask the jury to reconsider the issue, or alternatively order a new trial. Answer choice A is incorrect. Generally a court, unless ruling on a party's renewed motion for judgment as a matter of law, may not order the entry of a judgment that is contrary to a jury's general verdict. However, when a jury is instructed to return a general verdict as well as answer special interrogatories and the jury's answers to special interrogatories are inconsistent with each other, the court cannot order the entry of the jury's general verdict. The court must either direct the jury to reconsider its answers or order a new trial. Answer choice B is incorrect. When a jury is instructed to return a general verdict as well as answer special interrogatories and the jury's answers to special interrogatories conflict with each other, the court does not have the discretion to order the entry of the jury's general verdict—it cannot enter judgment. Answer choice C is incorrect because Federal Rule of Civil Procedure 49 specifically permits the use of special interrogatories along with a general verdict.

A loan shark instructed a henchman to either collect a debt from a borrower or shatter his kneecaps as a message to other borrowers of the importance of timely repayment of their loans. The henchman, mistaking the borrower's twin brother for the borrower, shattered the twin's kneecap when the twin refused to pay back a debt he had never incurred. The twin died from the shattered kneecap when a fragment of bone severed an artery and he bled to death. Is the loan shark likely to be found guilty of attempted murder of the borrower?

No, because the loan shark did not intend for the shattering of the borrower's kneecaps to result in his death. Although murder is a malice crime that can be satisfied by an intent to do serious bodily harm, attempted murder is a specific-intent crime. Here, the loan shark had the intent to do serious bodily harm to the borrower by subjecting him to a shattered kneecap. However, the loan shark did not possess the intent to kill the borrower. Therefore, even though the loan shark took a substantial step towards the murder of the borrower in directing his henchman to shatter the borrower's kneecaps, the loan shark is likely not guilty of attempted murder because attempt requires specific intent. Answer choice A is incorrect. A mistake of fact can be a defense to a malice crime such as murder if the mistake is a reasonable one. A mistake of fact can also be a defense to a specific-intent crime such as attempted murder even if the mistake of fact is an unreasonable one. Mistaking the borrower's twin for the borrower is likely a reasonable mistake. More importantly, the mistake did not result in the death of the borrower or even an attempt being made on his life, but instead in the death of the borrower's twin brother. Answer choice C is incorrect. Generally, vicarious-liability crimes are limited to regulatory crimes, and punishment is limited to fines. Instead, if the loan shark is liable for the actions of his henchman, it will be because the loan shark is an accomplice who is also responsible for the crime committed at his direction by his henchman. Answer choice D is incorrect because the doctrine of transferred intent can make the loan shark and his henchman liable for the murder of the borrower's twin brother, but it is inapplicable with respect to the borrower himself. Further, attempt is a specific-intent crime.

A man well over six feet tall decided to rob a convenience store. The man walked into the store, handed the store clerk, who was five feet tall, a bag and told her to put all the money from the cash register in the bag or he would kill her. The clerk, believing the man due to his large stature even though he did not display a weapon, put the money from the register into the bag. A police officer entered the store just as the clerk finished. The man, unsettled by the police officer's presence, left the bag and walked out of the store before the stunned clerk could alert the police officer. The man has been charged with robbery. Can he properly be convicted of this crime?

No, because the man did not take the bag. Robbery is larceny from the person or presence of the victim by force or intimidation. Larceny is the (i) trespassory (ii) taking and carrying away (iii) of the personal property of another (iv) with the intent to steal. Here, it can be argued that the man took the money by compelling the clerk to place the money in his bag. However, in addition to taking, larceny requires the defendant to carry the property away. Although this element of the crime, also known as asportation, requires only a slight movement of the property, the man did not take the bag from the clerk and therefore did not carry the money away. Consequently, because the man has not committed larceny, he cannot be convicted of robbery. Answer choice A is incorrect. Unlike burglary, which requires only that a defendant have the intent to commit a felony at the time that the defendant breaks into and enters the dwelling but does not require the burglar to actually commit that felony, robbery requires the defendant to commit larceny. Here, the man cannot be convicted of robbery because he did not commit larceny. Answer choice B is incorrect. The defendant must obtain the property of another by force or intimidation. The threat must be of immediate serious physical injury to the victim, a close family member, or other person present. Here, the difference in size between the man and clerk coupled with his threat to kill her is sufficient to constitute intimidation, especially as the clerk believed the man. However, intimidation of the victim is not enough—the defendant must also carry away the property. Answer choice D is incorrect because, although robbery does require the use of force or intimidation, it does not require the use or presence of a weapon.

A city had recently experienced a number of arsons in a residential area. Police arrived on the scene of a residential fire after the firefighters identified signs of an accelerant. The officers fanned out through the crowd of bystanders to ask whether any of them had seen anything suspicious that might suggest the work of an arsonist. Many bystanders simply declined to speak to the officers and walked away. However, one officer noted that a man in the crowd was acting nervous. As the officer approached the man, he could smell gasoline. The officer told the man he had a few questions for him. The man said he didn't really want to talk. As the man turned to leave, the officer mentioned that the firefighters had found evidence that the fire was started with gasoline. The man burst into tears and confessed to setting the fire, and the officer arrested him. The man was then given Miranda warnings. The man's attorney filed a motion to suppress the man's crime scene confession. Should the court suppress the man's confession to arson?

No, because the man was not in custody. An individual must be in custody to necessitate Miranda warnings. "Custody" requires the existence of conditions that would cause a reasonable person, under the totality of the circumstances, to believe that he is not free to leave. The questioning of a person at a crime scene does not constitute custody for the purposes of Miranda if the individual being questioned is free to leave. Here, the facts indicate that several bystanders did leave during the questioning process, and that the man himself even felt free to leave. Answer choice A is incorrect because, although a statement designed to elicit a response may be considered an interrogation for purposes of Miranda, that interrogation still must be custodial in order for Miranda warnings to be required. Answer choice B is incorrect because the man remained free to leave even after the officer continued to speak to him. Answer choice D is incorrect because interrogations must be custodial in order to require Miranda warnings, so whether the man was being interrogated is not the outcome determinative issue here.

A police officer believed that a particular house was being used to conduct drug deals. He observed people coming and going at all hours, although he did not see any drugs change hands. One afternoon, the officer observed a person drop a small bag on the porch when he was leaving the house. The person picked it up, but the officer thought that perhaps some residue remained. The officer went to the police station, got a trained drug-detection dog, and went to the house. He went up onto the porch with the dog, and the dog alerted him to the presence of drugs. Based on the dog's reaction, a search warrant was issued for the house. A subsequent search led to the discovery of various kinds of drugs. The owner of the house was arrested and charged with drug possession and distribution. The owner moved to suppress the drugs found, claiming that the use of the trained drug-detection dog constituted an illegal search. Should the court grant the owner's motion to suppress?

No, because the officer had probable cause to conduct a search based on his observations of the house. The use of a trained dog to sniff for the presence of drugs is a search if it involves a physical intrusion onto constitutionally protected property, including the curtilage of a home. In this case, the officer took the dog onto the porch of the house. The officer did not have a warrant, and no exceptions to the warrant requirement applied to his conduct. Therefore, the officer's actions constituted an illegal search, and the drugs found as a result of that search should be suppressed. Answer choice A is incorrect because, in the absence of a physical intrusion onto constitutionally protected property, the use of a drug-sniffing dog does not violate a reasonable expectation of privacy. Answer choice C is incorrect. Even if the coming and going of potential customers was enough to establish probable cause, the officer would still have to get a warrant to conduct a search of the house, including the curtilage of that house, unless a warrant exception applied. Answer choice D is incorrect. In the absence of a physical intrusion onto constitutionally protected property, the use of a drug-sniffing dog does not violate a reasonable expectation of privacy. However, the use of a trained dog to sniff for the presence of drugs does constitute a search if, as in this case, it involves a physical intrusion onto constitutionally protected property.

An off-duty police officer was having a drink with his wife in a bar. A man walked by their table, and made a lewd comment about the wife as he passed. The officer jumped to his feet and asked the man to repeat what he had said. The man, who was obviously intoxicated, repeated the comment. The officer then pushed the man, causing the man to fall backwards. The man rose to his feet, pulled a knife from his waistband, and walked toward the officer. As the man lunged at the officer with the knife, the officer pulled his gun from his waistband and shot the man in the leg. Although the officer did not intend to fire a lethal shot, the defendant had a disease that prevented his blood from properly clotting, and he died from blood loss due to the injury. Is the officer likely to be convicted of homicide?

No, because the officer was entitled to use self-defense under the circumstances. An initial aggressor gains the right to act in self-defense when an aggressor using nondeadly force is met with deadly force or the aggressor, in good faith, completely withdraws from the altercation and communicates this fact to the victim. In this case, the officer's nondeadly force was met with deadly force, and thus he was entitled to use self-defense. Answer choice A is incorrect because, if the officer had not been justified in using self-defense, he could be held liable for the death of the man because he intended to do grievous bodily injury to the man. Answer choice C is incorrect because the officer did not need to communicate his intent to withdraw in this instance because his use of nondeadly force was met with deadly force. Accordingly, the officer was entitled to use self-defense. Answer choice D is incorrect because the officer was entitled to use self-defense and thus would not be held liable for the man's death despite his intent.

After an arson suspect was indicted and awaiting trial, the police found a potential eyewitness to the crime. Through an acquaintance, the suspect learned that the witness was being brought to the police station for a photo array. The suspect did not tell his attorney or object to the photo array. The eyewitness identified the suspect in the photo array, and the prosecution plans to use the identification at trial. Defense counsel objects, on the grounds that he should have been present during the identification. Did the post-indictment photo array violate the suspect's constitutional rights?

No, because the photo array was not an in-person identification. The Sixth Amendment protects an accused's right to "the assistance of counsel for his defense." The Sixth Amendment right to counsel applies at all critical stages of a prosecution, after formal proceedings have begun. The right automatically attaches when the State indicts the defendant or brings formal charges. However, the Sixth Amendment right to counsel generally does not apply to certain "noncritical stages," such as photo identifications. Because the defendant is not present at a photo identification, the Sixth Amendment does not guarantee counsel at such a noncritical stage. Answer choice A is incorrect because there is no need to invoke the right to counsel guaranteed by the Sixth Amendment. It automatically attaches upon an indictment or formal charge and applies at all critical stages of a prosecution. However, it does not apply to a photo identification, which is considered a noncritical stage. Answer choice C is incorrect because the Fifth Amendment right to counsel must be invoked and is only applicable to custodial interrogations. Answer choice D is incorrect because, as explained above, photo identifications are not considered critical stages to which the Sixth Amendment attaches, even if they are conducted post-indictment.

A defendant was arrested for driving while intoxicated and was read his Miranda rights before officers placed him in a squad car and drove him to the police station for booking. When the officers in the car asked the defendant what happened, the defendant said he refused to tell them anything until he saw a lawyer. At the station, the officers asked the defendant to take a breathalyzer test as part of the booking process. The defendant repeatedly refused, but the officers were persistent. Eventually, the defendant consented and took the breathalyzer. The defendant was placed in jail, and when he woke up sober the next day, he met with his lawyer. At the defendant's trial, the defense attorney seeks to suppress the breathalyzer result as a violation of the defendant's privilege against self-incrimination. Should the court suppress the breathalyzer result on this ground?

No, because the privilege against self-incrimination only protects testimonial evidence.The Fifth Amendment right against self-incrimination, as applied to the states by the Fourteenth Amendment, applies only to testimonial evidence, and not to nontestimonial physical evidence. Because the breathalyzer result is not testimonial in nature, the privilege against self-incrimination will not apply here. Answer choice A is incorrect. Although this reasoning would allow the breathalyzer result to be admissible under the Fourth Amendment, it is not applicable to the privilege against self-incrimination. Answer choice C is incorrect because the breathalyzer result will not be considered an involuntary confession because it is not a testimonial statement. Answer choice D is incorrect because the defendant's specific, unambiguous statement asserting his desire to remain silent will not affect the admissibility of the nontestimonial breathalyzer result.

A state raised its minimum wage to $15 per hour. A federal post office in the state pays some of its employees the federal minimum wage of $7.25 an hour. Can the post office be prosecuted for failing to adhere to the new state minimum wage?

No, because the state cannot regulate the federal government absent congressional consent. The states have no power to regulate the federal government unless Congress permits the state regulation or unless the state regulation is not inconsistent with existing federal policy. Here, the state's minimum wage statute penalizes the behavior of an agency of the federal government, which is immune from that regulation under the concept of sovereign immunity. Answer choice A is incorrect. When federal and state governments legislate in the same area, the Supremacy Clause provides that federal law supersedes conflicting state law. However, there is no conflict here that will trigger preemption. A state law conflicts with federal law if it allows conduct that is forbidden by the federal law or makes it impossible (or nearly so) to comply with both. Here, an employer can easily comply with both the state and federal law when it obeys the state law. Therefore, preemption does not apply. Answer choice C is incorrect because the states have no power to regulate the federal government unless Congress permits the state regulation or unless the state regulation is not inconsistent with existing federal policy. This immunity is not limited to state taxation, and even some state taxation may be permitted. Answer choice D is incorrect. Even though the post office employees are working within the state, the employees are working for an agency of the federal government. Although the employees' wages can be subject to state income tax, the federal government as an employer is not subject to the state's minimum wage.

A state statute was recently enacted prohibiting any grocery store or market within one mile of a school from displaying any posters advertising items for sale. The goal of the statute was to discourage minors from illegally purchasing alcohol or soliciting others to purchase alcohol for them after seeing posters advertising that it was for sale. Before the regulation, many affected stores that did not sell alcohol also advertised in this manner, and the stores that did sell alcohol often advertised non-alcoholic items for sale with posters as well. Is the statute constitutional?

No, because the statute was not the least restrictive available means to achieve the government's goal. Answer choice A is correct. Commercial speech is entitled to intermediate First Amendment protection when it concerns lawful activity and is neither false nor misleading. Restrictions on such speech are valid if the regulation is narrowly tailored to serve a substantial governmental interest. In this context, narrowly tailored does not mean the least restrictive means available; rather, there must be a "reasonable fit" between the government's ends and the means chosen to accomplish those ends. Here, the ban on all posters is not a reasonable means to achieve the government's goal of discouraging minors from illegally acquiring alcohol. Although the governmental interest here is substantial, the ban on all posters does not directly or efficiently advance this interest. Answer choice B is incorrect because, in the context of regulating commercial speech, "narrowly tailored" does not mean the least restrictive means available; the regulation must merely be reasonable and proportional to the goal of the regulation. Answer choice C is incorrect because commercial speech does receive a measure of First Amendment protection. Therefore, this answer choice misstates the law. Answer choice D is incorrect because it states the rational basis test, and commercial speech is subject to more protection than that.

A wife discovered that her husband had been having an affair with her best friend for many months. Enraged, the wife decided to invite her best friend over for poolside cocktails, spike her drinks with a strong sedative, and then push her into the pool to drown after she passed out. The best friend agreed to come over, but because the wife was extremely nervous, she took ten times the recommended amount of her prescribed anti-anxiety medication. The best friend arrived to find the wife acting erratically, clearly under the influence of the drugs. However, the wife managed to add the sedative to the best friend's cocktails, and when she passed out, the wife pushed her into the pool. The best friend drowned. The police eventually arrested and charged the wife with first-degree murder. The jurisdiction defines first-degree murder as a deliberate and premeditated unlawful killing of another human being. Does the wife have a valid intoxication defense to the first-degree murder charge?

No, because the wife formed the intent to kill before she became intoxicated. Answer choice D is correct. Voluntary intoxication is the intentional taking of a substance known to be intoxicating. Voluntary intoxication is not a defense to a specific-intent crime when the intent was formed before intoxication. Here, the wife committed first-degree murder, a specific-intent crime, because it was premeditated and deliberate. However, because she formed the intent to kill her best friend before she took the anti-anxiety drugs, voluntary intoxication is not a defense. Answer choice A is incorrect. Although voluntary intoxication can be a defense to specific-intent crimes, here, the wife formed the requisite intent before her intoxication. Answer choice B is incorrect. Involuntary intoxication can give rise to an insanity defense if the requirements for that defense are met. However, in this case, the wife voluntarily intoxicated herself and formed the intent to kill before becoming intoxicated. Answer choice C is incorrect. Intoxication can be caused by any substance (e.g., alcohol, drugs, or prescription medicine), including a prescribed anti-anxiety medication.

A witness was subpoenaed to testify at a grand jury proceeding investigating allegations of a large-scale identity-theft operation. The witness was accompanied by his attorney, but the prosecutor refused to let the witness' attorney into the grand jury room. The witness also asked to call another individual as a witness to corroborate his alibi, but the prosecutor denied this request as well. After speaking with his attorney outside the grand jury room, the witness decided to testify. The witness was later indicted by the grand jury for identity theft. The witness has moved to dismiss the indictment as violating his Fifth Amendment right to counsel and his Sixth Amendment right to present witnesses. Should the court dismiss the indictment?

No, because the witness was not denied any constitutional rights.A grand jury witness has no Fifth Amendment right to counsel in the grand jury room. The witness may, however, consult with an attorney outside the grand jury room. Additionally, a witness has no Sixth Amendment right to present witnesses or to introduce evidence at a grand jury proceeding. Therefore, this fact pattern does not suggest that any of the witness's constitutional rights have been violated. Answer choice A is incorrect because a witness testifying before a grand jury does not have a right to have counsel present in the grand jury room. Answer choice B is incorrect because a witness has no Sixth Amendment right to present or confront witnesses or to introduce evidence at a grand jury proceeding. Answer choice C is incorrect because, although the witness did waive his Fifth Amendment right against self-incrimination by testifying, the witness's Fifth Amendment right to counsel was not violated and he did not have a Sixth Amendment right to present witnesses to the grand jury.

The police, after witnessing a drug sale, arrested the seller. The police brought the suspect to the station, where they properly advised him of his Miranda rights. The suspect immediately asked for his attorney. The police did not question the suspect while they waited for the attorney to arrive, but an officer did remain with the suspect. When the officer escorted the suspect to the bathroom, the suspect saw a drug dealer from the neighborhood being led to a different interrogation room. The suspect told the officer that the police should not believe anything the drug dealer said about him, because the dealer was jealous of the suspect ever since he had taken over some of the drug dealer's turf. When the prosecution sought to introduce these statements at trial, the suspect moved to suppress them. Are the statements likely to be suppressed?

Once a custodial suspect invokes his right to counsel, all interrogation must stop until counsel is present. However, if the suspect makes any voluntary or spontaneous statements, even after invoking the right to counsel, those statements are admissible. In this case, the police did not question the suspect after he invoked his right to counsel. The suspect initiated communication when he voluntarily made statements about the drug dealer. Consequently, the suspect's statements are admissible. Answer choice A is incorrect because the police may not question a suspect after he invokes his right to counsel; that request must be scrupulously honored. Answer choice C is incorrect because, although the suspect clearly remained in custody, his statements were not made in response to police interrogation; the mere presence of an officer in the room with a suspect is not coercive. Answer choice D is incorrect because, although the suspect asserted his Fifth Amendment right to counsel, the suspect's subsequent statements were voluntarily made rather than in response to police interrogation.

An employee at a fast food restaurant was held up at gunpoint by two criminals. The criminals ordered the employee to empty the cash register. The employee did so, and handed over $55 in cash to them. The criminals took the money and ran off before the police arrived. After they left, a restaurant customer immediately approached the employee to see if she was all right. The employee was visibly shaken, but she reached under the counter, pulled out a bank deposit envelope, set it directly in front of her on the counter, and told the customer that luckily she had recently emptied the cash register and put it into the envelope. The customer grabbed the envelope from the counter and ran out of the restaurant. The employee was so stunned that she was unable to react. The police eventually caught the customer, and charged him with robbery. Is the customer guilty of robbery?

Robbery is larceny from the person or presence of the victim by force or intimidation. Larceny is the trespassory taking and carrying away of the personal property of another with the intent to permanently deprive the person of the property. Here, the grabbing of the bank deposit envelope does not qualify as force or intimidation. The customer grabbed the envelope so quickly that the employee could not react, and the employee was not touching the envelope, so there was neither force nor intimidation needed to take it. Note that this does not mean the customer would not be guilty of any theft crime, just not the crime of robbery. Answer choice A is incorrect because although the customer had the intent to permanently deprive the employee of the envelope, this is just one prong of the rule for robbery. There is no force or intimidation, so the employee cannot be guilty of robbery. Answer choice B is incorrect because the force used by other parties (i.e., the criminals) does not satisfy the requirement of force or intimidation for the customer to be found guilty of robbery. Answer choice D is incorrect because grabbing the envelope off of the counter, in the employee's presence, qualifies as a taking.

A federal study found that almost all of the accidents on a particular state's highways were caused by out-of-state residents. Congress passed a law requiring the payment of tolls at all of the state's borders. The law also required that any driver with an out-of-state driver's license pay an additional toll and leave a credit card number for the state to use in the event that the out-of-state driver caused an accident on one of the state's roads. Several out-of-state motorists have challenged the constitutionality of the law. Which of the following provides the best ground for challenging the constitutionality of the law?

The Due Process Clause of the Fifth Amendment. The Due Process Clause of the Fifth Amendment applies against the federal government, and provides that "no person shall be...deprived of life, liberty, or property, without due process of law." The Due Process Clause contains a substantive component that guarantees certain fundamental rights, including the right to travel among the states. This includes the right to enter one state and leave another and to be treated as a welcome visitor. Because this law impinges on a fundamental right, it is subject to strict scrutiny. Answer choice B is incorrect because the Fourteenth Amendment does not apply to the federal government, although the Due Process Clause of the Fifth Amendment includes the rights guaranteed by the Equal Protection Clause of the Fourteenth Amendment. Answer choice C is incorrect because the Commerce Clause does grant Congress the power to regulate interstate travel. Answer choice D is incorrect because the Privileges and Immunities Clause prevents one state from discriminating against citizens of another state; it does not apply to the federal government.

A state statute provides that local county governments within the state may adopt an ordinance that prohibits the sale of alcoholic beverages within the county. A local county government, in accord with this statute, adopted a resolution banning the sale of alcoholic beverages within the county with an alcoholic content in excess of 10 percent. Which of the following provides the best support for the constitutionality of this ordinance?

The Twenty-First Amendment, in addition to repealing prohibition, specifically gives states the authority to prohibit the transportation or importation of alcoholic beverages into the state for delivery or use within the state. Answer choice A is incorrect because the Dormant Commerce Clause, rather than supporting a state's regulation of alcoholic beverages, serves as a check on such regulation. Answer choice B is incorrect. The Tenth Amendment generally reserves to the states the powers not delegated to the federal government by the Constitution or prohibited to the states. However, the Twenty-First Amendment specifically grants to the states the power to prohibit the transportation or importation of alcoholic beverages in the state for delivery or use within the state. Answer choice C is incorrect because the Equal Protection Clause, rather than supporting a state's regulation of alcoholic beverages, serves as a check on such regulation.

A manufacturer, on its own land, built a factory to produce a product. The factory contained the heavy machinery needed to produce the product. Several years later, the owner of adjoining land excavated a site on its own land in order to place an underground storage tank. The excavation at this location, although done without negligence, caused the part of the manufacturer's factory that housed the heavy machinery to subside. Both the manufacturer with regard to the factory and the adjacent landowner with regard to the excavation complied with all governmental regulations. The manufacturer has sued the adjacent landowner for damages to its factory attributable to the landowner's excavation. For whom is the court likely to rule?

The adjacent landowner, because the subsidence occurred without negligence on its part. A landowner has a right to lateral support from adjoining land. When adjoining land is in its natural state (i.e., undeveloped), a landowner who excavates on his own land is strictly liable for any damage to the adjoining land caused by the excavation. If the adjoining land has been improved (i.e., is not in its natural state), the excavating landowner is strictly liable for any damage caused by the excavation only if the land would have collapsed in its natural state (regardless of the improvement). If the improvement contributed to the collapse, then the adjoining landowner may recover only if the excavating landowner was negligent. Here, the manufacturer's land was not in its natural state; the manufacturer had constructed a factory on the land. Moreover, the only part of the factory that suffered damages was the part that had housed the heavy machinery, which indicates that the land would likely not have subsided in its natural state. Consequently, because the subsidence occurred without negligence on the part of the adjacent landowner, the court is likely to rule in favor of the adjacent landowner. Answer choice A is incorrect because liability based on the duty to provide lateral support is not predicated on a landowner's obligation not to damage existing improvements on adjoining land, but instead on a landowner's duty not to negligently cause damage to such structures that contribute to the subsidence. When the landowner's actions are not negligent, the owner of adjacent land cannot recover for the damage caused to such structures. Answer choice B is incorrect. A landowner is strictly liable for damages to adjoining land caused by excavation if the adjoining land is in its natural state, but the land here was not as it had been improved by the construction of a factory. In addition, if the adjoining land has been improved (i.e., is not in its natural state), the excavating landowner is strictly liable for any damage caused by the excavation only if the land would have collapsed in its natural state (regardless of the improvement). Here, there are no facts to indicate the land would have collapsed in its natural state. Answer choice D is incorrect. A landowner still owes a duty of lateral support with regard to adjoining land even though the landowner's actions were performed solely on his land.

A 60-year-old driver received a traffic citation for his failure to use the headlights on his automobile after dark. He pled not guilty and requested a trial, at which he was found guilty. He requested that, in lieu of paying a fine, he be required to attend the driving school course that was offered by the municipal court for certain traffic offenses. The court program offering driving school in lieu of fine was rationally limited to drivers between the ages of 15 and 18 years old because they were the most likely to modify their driving habits and thereby benefit from the school. Based on this reasoning, the judge denied the driver's request solely because he did not qualify for driving school due to his age. The driver has challenged the judicial refusal to admit him into driving school as a violation of the Equal Protection Clause of the Fourteenth Amendment to the U.S. Constitution. Of the following reasons, which best supports the rejection of the driver's constitutional challenge?A 60-year-old driver received a traffic citation for his failure to use the headlights on his automobile after dark. He pled not guilty and requested a trial, at which he was found guilty. He requested that, in lieu of paying a fine, he be required to attend the driving school course that was offered by the municipal court for certain traffic offenses. The court program offering driving school in lieu of fine was rationally limited to drivers between the ages of 15 and 18 years old because they were the most likely to modify their driving habits and thereby benefit from the school. Based on this reasoning, the judge denied the driver's request solely because he did not qualify for driving school due to his age. The driver has challenged the judicial refusal to admit him into driving school as a violation of the Equal Protection Clause of the Fourteenth Amendment to the U.S. Constitution. Of the following reasons, which best supports the rejection of the driver's constitutional challenge?

The age limitation on the driving school option is rationally related to a legitimate governmental interest of encouraging safe driving habits. Age discrimination does not provoke heightened scrutiny. Therefore, laws and other governmental actions classifying on the basis of age are reviewed under the rational basis standard. Here, the facts establish that there is a rational reason for limiting the driving school option to youthful drivers. Answer choice B is incorrect because, while age is not a suspect class that requires heightened scrutiny, the Equal Protection Clause does require that governmental discrimination based on age satisfy the rational basis test. Answer choice C is incorrect because a judicially administered program constitutes governmental action that is subject to the Equal Protection Clause. Answer choice D is incorrect because a program offered by a municipal court qualifies as state action for purposes of the Equal Protection Clause.

Congress enacted legislation that made funding available to the states to provide services for children with special needs. The act conditions the funding on the states' meeting a variety of conditions designed to ensure that children with special needs received adequate services. In addition, the act grants parents of a child with special needs a right of action against a local school board for the failure to provide adequate services. The act permits the parents to recover the cost of tuition at a private school that meets the child's needs, together with "attorney fees and costs." The act is silent as to whether expert fees are considered costs. The parents of a child with special needs successfully pursued an action against the local school board under this statute. The court awarded the parents the appropriate damages, as well as attorney's fees and costs, including expert fees. Of the following, which is the most likely constitutional basis on which the local school board can challenge the court's award?

The awarding of expert fees is unconstitutional because conditions imposed on states by Congress in the exercise of its spending power must be set out unambiguously. While Congress has the power under the spending power to spend for the general welfare and may impose conditions on the receipt of an appropriation by a state, such conditions must be set out unambiguously to be enforceable. Since expert fees were not unambiguously included in the amounts that may be recovered by the parents of a child with special needs under this statutory cause of action, these fees cannot be recovered. Answer choice A is incorrect. The Tenth Amendment provides that all powers not assigned by the Constitution to the federal government are reserved to the states, or to the people. However, the federal government has very broad authority under the spending power to spend for the general welfare, making state power rarely exclusive. Therefore, because states do not have exclusive power over education, this answer choice is incorrect. Answer choice C is incorrect. The spending power has been interpreted very broadly. Congress has the power to spend for the "general welfare"—i.e., any public purpose—not just to pursue its other enumerated powers. Therefore, this answer choice misstates the law. Answer choice D is incorrect. The Eleventh Amendment is a jurisdictional bar that prohibits the citizens of one state from suing another state in federal court. While the Supreme Court has expanded the amendment's reach to preclude citizens from suing their own state in federal court as well, this amendment does not apply to suits brought against local governments and their entities, such as school boards. Consequently, the provision in the act providing parents the right to sue their local school board does not violate the Eleventh Amendment.

A baker who had a contract for blueberry muffins tried unsuccessfully to negotiate with a blueberry farmer to purchase blueberries. The following evening, the baker waited until the farmer left her roadside stand and then entered the farmer's adjacent blueberry fields, and picked 25 quarts of blueberries. As the baker was loading the blueberries into his van, the baker had second thoughts about what he had done. Realizing the farmer would have no problem selling the blueberries at his stand, the baker carried the buckets filled with blueberries to the front of the farmer's stand. A few moments later, he was apprehended and charged with larceny. Of the following, which would provide with baker with the best defense? Question 7316

The baker had picked the blueberries himself. Answer choice D is correct. Larceny is the (i) trespassory (ii) taking and carrying away (iii) of the personal property of another (iv) with the intent to permanently deprive that person of the property. The taking of real-property items (e.g., unharvested crops) is not larceny when the defendant's act of severance occurs immediately before the carrying away of the real-property items. The act of harvesting fruit constitutes the severance of real, rather than personal property. Here, because the baker had picked the blueberries himself, he had effected their severance from the farmer's field and the blueberries the baker picked are considered to be real rather than personal property. Answer choice A is incorrect because, had the blueberries been personal rather than real property, the baker would have completed the crime of larceny prior to beginning to return the blueberries to the farmer's stand. He had taken and carried away the farmer's blueberries with the intent of using those blueberries. Answer choice B is incorrect. Although larceny does require the defendant to possess the intent to permanently deprive the victim of the property, larceny does not require that the victim be permanently deprived of the property. Here, the baker had the intent to use blueberries to make muffins at the time that he picked and transported the blueberries to his van. Answer choice C is incorrect. Although larceny usually results in the victim's loss of property, larceny does not require the owner of the stolen property to have suffered a loss. Here, even though the farmer could have sold the blueberries that the baker picked at her farm stand presumably without suffering a loss, this is not a defense to the baker's commission of the crime of larceny.

A woman placed an online order to purchase a unique tea kettle from a kettle manufacturer as a gift for her brother. The kettle was square-shaped and had two spouts. The kettle's shipping box stated the following: "Caution! The enclosed kettle is a novelty item and should not be used to pour boiling water. Use with boiling water may result in steam burns." Before using the kettle, the brother read some online reviews of the kettle. Many reliable reviews stated that steam burns were very common when using the kettle, because steam would escape from the second spout if the kettle was not held properly. However, many reviews suggested that the kettle could be used safely if it was held in a particular way. The first time the brother used the kettle, he carefully followed the instructions from the reviews on how to hold the kettle when filled with boiling water. However, while he was pouring the boiling water from one spout, scorching steam escaped from the other spout and burned the brother's skin. The jurisdiction applies the common-law rules for contributory negligence and assumption of the risk. If the brother files a strict-products-liability suit against the manufacturer, what is the manufacturer's best defense?

The brother assumed the risk of being burned. Under the doctrine of strict products liability, a seller of a product is liable for personal injuries caused by that product, even in the absence of fault, if the product was defective, the defect existed at the time the product left the defendant's control, and the defect caused the plaintiff's injury when used in an intended or reasonably foreseeable way. However, a voluntary and knowing assumption of the risk is a complete bar to recovery in contributory-negligence jurisdictions if a plaintiff is aware of the danger and knowingly exposes himself to it. Here, the brother knew that steam burns were possible if the novelty kettle was used with boiling water, but decided to use the kettle with boiling water anyway. Accordingly, assumption of the risk provides the manufacturer with the strongest defense to the brother's strict-products-liability suit. Answer choice B is incorrect. Mere misuse will not constitute a defense to a strict products liability claim if the misuse is reasonably foreseeable. Here, the fact that the brother misused the kettle would likely be a foreseeable misuse of the kettle, especially based on the number of online reviews indicating users had used the kettle with boiling water. Therefore, this argument would not protect the manufacturer from liability. Answer choice C is incorrect. Anyone foreseeably injured by a defective product may bring a strict-liability action. Appropriate plaintiffs include not only purchasers, but also other users of the product. There is no requirement that the brother must be a purchaser in order to recover. Answer choice D is incorrect. In a contributory-negligence jurisdiction, the plaintiff's negligence generally is not a defense to a strict-products-liability action when the plaintiff negligently failed to discover the defect or misused the product in a reasonably foreseeable way. Ordinary contributory negligence by the plaintiff will not bar recovery based on strict products liability. Therefore, the manufacturer will only have a chance to succeed in defending against this action if it argues that the brother assumed the risk of steam burns.

The owner of a rare manuscript and a collector entered into a written agreement for the sale of the manuscript at a price of $7,500, which was to be paid upon delivery of the manuscript on a specific day. When the owner tendered the manuscript on the specified date, the collector refused to accept it or to pay for it. The owner tried in good faith, by commercially reasonable means, to sell the manuscript, but he was unsuccessful. He subsequently brought suit against the collector for specific performance of the contract. For whom should the court rule?

The collector, because damages are an adequate remedy for the owner. In general, specific performance is not available as a remedy for breach of a contract when damages are an adequate remedy for the nonbreaching party. Because the seller in a sale-of-goods contract typically receives money from the buyer in exchange for the goods, damages are an adequate remedy for the seller, as they would be in this instance for the manuscript owner. Answer choice A is incorrect. Although the collector did not take the manuscript when the owner tendered it to the collector on the date specified in their contract, the collector's refusal to pay for the manuscript at that time, as specified in the contract, constituted a breach of the contract. Answer choice C is incorrect. Although the owner has been unsuccessful in his attempts to sell the manuscript despite doing so in good faith and in a commercially reasonable manner, this failure does not prevent damages from being an adequate remedy for him. It simply means that his damages as measured by his inability to resell the manuscript would be the contract price of the manuscript. Answer choice D is incorrect. Although the Uniform Commercial Code (UCC) lists specific performance as a remedy for the buyer of goods, the UCC does not list specific performance as a corresponding remedy for the seller of goods. As noted with respect to answer choice B, damages are an adequate remedy for a seller who, pursuant to the contract, is to receive money from the buyer for the goods.

State law contains a variety of provisions regarding the homeschooling of a child by the child's parent or guardian, including specific subjects that must be taught; the number of hours of educational instruction the child must receive on a yearly basis; certification that the instructor has at least a high school diploma or its equivalent; and notification to the superintendent of the child's public school district of the parent's or guardian's intent to homeschool the child. The parents of an elementary school-aged child have challenged these requirements in federal court as violating their due process rights regarding their child's education. How should the court rule on the parents' challenge to these requirements?

The court must uphold them as a valid exercise of the state's police powers, because a state may compel a child to attend public school. While parents do enjoy a fundamental right to make decisions regarding the care, custody, and control of one's children, including the right to privately educate one's child outside the public school system, that right is subject to reasonable educational standards imposed by the state. Consequently, the court must strike down any requirements that the court finds impose unreasonable educational requirements on the homeschooling of children. Answer choice A, which fails to note that the parental right to educate their children privately may be subject to reasonable educational standards imposed by the state, is incorrect. Answer choice C is incorrect because, although a state may generally exercise its police power in any way that is not arbitrary nor irrational, a state may not require a child to attend public school when a parent or guardian chooses to educate the child at home. Answer choice D is incorrect because unreasonable educational standards imposed by the state will not be upheld merely because education is an area of state concern.

The Department of Defense entered into a contract with a state-owned weapons manufacturer. The contract called for the production of defense equipment to be used by the military. The Department of Defense agreed to purchase all of the defense equipment produced by the state-owned weapons manufacturer for a period of one year. Each purchase made by the Department of Defense during that one-year period was subject to taxation pursuant to state law. The Department of Defense filed suit in federal court challenging the constitutionality of the state tax applied to its purchases under the contract. Is the Department of Defense subject to the state tax for its purchase?

The federal government and its instrumentalities (such as a national bank) are immune from taxation by the states. In this case, the Department of Defense, part of the federal government, is immune from taxation by the state for any purchases made pursuant to the defense contract. Answer choice A is incorrect. The mere fact that the contract involved military defense is not enough to result in immunity from state taxation. The key fact here is that the Department of Defense is the purchaser under the contract, and thus it is immune from the state tax. Answer choice C is incorrect. Although the power to tax is an essential incident of state sovereignty, that power is limited when applied to the federal government. Answer choice D is incorrect. The Department of Defense did not consent to state taxation merely by entering into a contract for defense equipment from a state-owned manufacturer. It is immune from state taxation regardless of whether the other party to the contract was the state or a private entity.

A man was jealous of his brother's good fortune. Together with his best friend, the man formed a plan to take some valuable artwork from the brother's home. According to the plan, the friend would rent a van that they would use to drive to the brother's house. The man and his friend would then break into the brother's house, take the paintings, and load them into the van. The next week, the friend rented the van, picked up the man, and drove to the brother's house. When they arrived at the brother's house, the friend had a change of heart and told the man that he could not go through with breaking into the house. The man decided to go ahead with the plan on his own, after the friend ran off. The following day, the friend voluntarily went to the police station and confessed to the plan. Which of the following statements best describes the friend's criminal liability under the majority rule?

The friend is guilty of conspiracy and of larceny as an accomplice.Conspiracy is an agreement between two or more persons to accomplish an unlawful purpose with the intent to accomplish that purpose. Under the majority rule, a conspiracy does not exist until an overt act has occurred, and withdrawal is possible between the date of the agreement and the commission of the overt act. In order to withdraw, notice must be communicated to the other co-conspirators, or the police must be advised of the existence of a conspiracy in a timely manner. Upon completion of the overt act (here, renting the van), the conspiracy is formed, and withdrawal is no longer possible. Under the majority and MPC rule, an accomplice is a person who, with the purpose of promoting or facilitating the commission of the offense, aids or abets a principal prior to or during the commission of the crime. An accomplice is responsible for the crime to the same extent as the principal. To legally withdraw (and therefore avoid liability for the substantive crime), the accomplice must repudiate prior aid or do all that he can to negate the prior assistance before the crime is put into motion. A mere change of heart after the crime is put into motion is ineffective, as notification to the legal authorities must be timely and directed toward preventing others from committing the crime. In this case, the friend did not effectively withdraw as an accomplice or co-conspirator until after the man had the van in front of his brother's house, and the friend did not immediately notify the police so as to prevent the crime. Accordingly, he would be guilty of larceny as both co-conspirator and accomplice. Answer choice A is incorrect because the friend would be guilty of larceny, as he entered into a conspiracy and rented the van, picked up the man, and drove to the brother's house before he attempted to withdraw. Answer choice B is incorrect because the friend did enter into a conspiracy with the man. Answer choice C is incorrect because the friend aided the man in committing the crime by renting and providing him with the van.

A man decided to siphon gasoline out of his neighbor's recreational vehicle (RV) into a milk jug. When he had almost filled his jug, his sweater discharged a spark of static electricity and ignited the gasoline. Moments later the RV was blazing. The man was charged with larceny and arson. At trial, the judge instructed the jury that if the jury found that the man was guilty of larceny of the gasoline and that the damage to the RV was the result of the larceny, then the jury should also find him guilty of arson. The man was convicted on both counts. The man appealed his conviction for arson. The man was convicted in a jurisdiction that follows the common-law definition of arson and recognizes the neighbor's RV as a dwelling. What is the man's strongest argument that the conviction should be overturned?

The jury instruction was incorrect because the jury instruction improperly described the required mental state for arson. The jury instruction was incorrect because arson is a crime of malice at common law. Therefore, an instruction making the man liable for arson if it was an unintended result of his larceny violated due process because it removes the prosecution's burden of proving the requisite mental state for arson of malice beyond a reasonable doubt. Answer choice A is incorrect because the crimes of larceny and arson do not merge. Answer choice B is incorrect because double jeopardy does not preclude conviction of two crimes, each with at least one different legal element, arising from a single act. Answer choice C is incorrect because the doctrine of transferred intent can apply to arson. Although the doctrine of transferred intent can apply to arson, it does not apply under these facts because the man's act of siphoning gasoline from his neighbor's RV did not directly result in the burning of the RV, rather it was the unintended spark that directly led to the fire.

A telecommunications company applied for a permit to construct a tower on land that the company owned to host a wireless communications signal. The local zoning board denied the permit based on the existing zoning of that land. Prior to seeking a special use variance, the company filed suit in federal court seeking a declaratory judgment that the zoning board's denial violated a federal law that prohibits a state or local government from effectively denying the provision of wireless services to the public. Of the following, which likely constitutes the local government's best constitutional defense against this suit?

The matter is not ripe for adjudication. Under the ripeness doctrine, which is grounded in part on the "case or controversy" requirement of Article III, Section 2, a federal court will not consider a claim before it has fully developed, as doing so would be premature, and any potential injury would be speculative. Here, because the company has failed to seek a special use variance, it is likely that court will find the company's claim is premature. Answer choice A is incorrect because whether the denial of a permit by the local zoning board constitutes an unconstitutional taking is irrelevant. The company's action is based on an alleged violation of a federal statute, not on an assertion that the denial has resulted in an unconstitutional taking. Answer choice C is incorrect. Although the Eleventh Amendment does bar certain suits against a state in federal court, when a state official, rather than the state itself, is named as the defendant in an action brought in federal court, the state official may be enjoined from enforcing a state law that violates federal law or may be compelled to act in accord with federal law despite state law to the contrary. Answer choice D is incorrect because the fact that a plaintiff is seeking a declaratory judgment as a form of relief does not violate the "case or controversy" requirement.

By warranty deed, a man conveyed land to his niece and nephew "as joint tenants with right of survivorship." A month later, the niece conveyed all of her real property interests to her daughter. The niece then died intestate. After the niece died, the nephew conveyed any interest he might have in the land to his son by quitclaim deed. There is no applicable statute, and the jurisdiction recognizes the common-law joint tenancy. Who holds valid title to the land?

The nephew's son and the niece's daughter. A joint tenancy exists when two or more individuals own property with the right of survivorship (i.e., upon the death of a joint tenant, the interest terminates and automatically goes to the surviving joint tenant(s)). A lifetime (inter vivos) transfer by a joint tenant of his property interest effects a severance of the joint tenancy, converting it to a tenancy in common. Here, the niece severed the joint tenancy when she conveyed her interest to her daughter, resulting in the daughter and the nephew having a tenancy in common. The nephew then conveyed his interest in the land to his son. Therefore, the nephew's son and the niece's daughter hold valid title to the land as tenants in common. Answer choice A is incorrect because the conveyance by the niece to her daughter severed the joint tenancy and granted the niece's daughter an interest in the land as a tenant in common. Answer choices C and D are incorrect because the niece's heirs do not have an interest in the land. The niece transferred her interest in the land to her daughter prior to death, and therefore had no interest in the land to leave her heirs upon her death.

A professor listed her car for sale on an online site. A buyer called and asked a series of questions, including the following, "I am not much of a music lover, but how does the stereo sound?" The professor replied that it sounded great. In reality, the stereo had short-circuited months earlier and blown out all the speakers in the process. Based on the conversation, the buyer entered into a written contract with the professor to purchase the car at a specified price. The contract provided for the buyer to pay for the car and take possession of it within three days. The next day, remorseful over her lie, the professor had the stereo and speakers restored to factory condition. On the day the buyer was supposed to take possession of the car, he learned from a student that the professor had lied about the condition of the stereo, and decided not to pay for the car. When the professor called the buyer later that day, the buyer told her he was not purchasing the car. The professor admitted her lie, and told the buyer that the stereo and speakers had been replaced. Nevertheless, the buyer refused to pay for the car. In a subsequent breach of contract suit, who will prevail?

The professor, because she repaired the stereo so that it conformed to the buyer's expectations. A misrepresentation is an untrue assertion of fact (not merely of law or opinion) that can make the contract void or voidable. Fraudulent misrepresentation requires proof of the following: (i) the misrepresentation is made knowingly and with intent to mislead the other party (i.e., it is fraudulent), (ii) the misrepresentation induced assent to the contract, and (iii) the adversely affected party justifiably relied on the misrepresentation. A fraudulent misrepresentation need not be material, and may make the contract voidable at the adversely affected party's option. The contract is no longer voidable, however, if, following a misrepresentation but before the deceived party has avoided the contract, the facts are cured so as to be in accord with the facts that were previously misrepresented. Here, although the professor made a fraudulent misrepresentation, she cured the misrepresentation by having the stereo restored to factory condition before the buyer avoided the contract. Therefore, the contract is binding on the buyer, and his failure to pay constitutes a breach. Answer choice B is incorrect because a fraudulent misrepresentation need not be material for the contract to be voidable. Moreover, a misrepresentation is considered material if it is likely to induce a reasonable person to manifest assent or the seller knows that the misrepresentation is likely to induce the buyer himself to manifest his assent. In this case, the misrepresentation would likely be considered material. Answer choice C is incorrect because a misrepresentation can be cured at any point before the adversely affected party avoids the contract. Answer choice D is incorrect because the buyer's right to terminate the contract was subject to whether the professor had first cured the defects underlying her misrepresentation

Using a path frequented by students, a college student decides to take a short cut through the back yard of a homeowner. The homeowner, who is unaware of the student's presence, is cleaning out the cage of a rattlesnake he is keeping at his house. The homeowner has carelessly allowed the snake to roam free. The snake, hidden from the student's view by a tree, is startled by the student as the student walks past and strikes the student. The student is seriously injured by the snakebite. The applicable jurisdiction permits the keeping of a rattlesnake as a pet. In a strict liability action by the student against the homeowner, who will prevail?

The student, because the homeowner possessed a wild animal. An owner of a wild animal is generally not strictly liable to a trespasser who is injured by the wild animal, except for injuries caused by a vicious watchdog. Here, the student was a trespasser and the homeowner's animal was not a vicious watchdog. Answer choice A is incorrect because, while the possessor of a wild animal generally is strictly liable to a licensee or invitee who is injured by the wild animal, the possessor is not strictly liable to a trespasser. Answer choice B is incorrect because the action brought by the student was based on strict liability, not negligence. Consequently, the homeowner's failure to act with reasonable care is irrelevant. Answer choice D is incorrect because, even though the homeowner's possession of the rattlesnake is legal, the rattlesnake is a wild animal. As such, the possessor of a wild animal may be strictly liable for harm caused by the animal, unless the individual harmed is a trespasser.

A single woman desperately wanted to have a baby, so she utilized the services of a sperm bank to get pregnant. The woman had a difficult pregnancy, and she suffered from postpartum psychosis after the birth of her baby son. She began seeing a psychiatrist, but rather than work through her postpartum psychosis, she spent the majority of her sessions talking about how she was convinced that the sperm bank had used her as part of a top-secret government experiment, and as a result of the experiment, she had given birth to Hitler's spawn. Although the woman had yet to harm her son, she grew more and more adamant in her belief that Hitler was the father of her son. Believing that she needed to save the world from another Hitler, she drowned her baby son. The woman was charged with murder, and her defense attorney raised an insanity defense. During the trial, the psychiatrist testified that the woman would not have killed her baby in the absence of postpartum psychosis, and that the woman did not think it was immoral to drown her son because she believed he was Hitler's spawn. The jurisdiction in which the court resides has adopted the M'Naghten test for insanity. What is the woman's strongest argument in support of an insanity defense?

The woman did not know that her act was morally wrong. Under the M'Naghten test, the defendant is not guilty if, because of a defect of reason due to a mental disease, the defendant did not know either the nature and quality of the act, or the wrongfulness of the act. Here, the woman suffered from postpartum psychosis, and she did not understand that it was morally wrong to drown her son. Answer choice A and D are incorrect because neither of these choices are supported by the facts. The woman understood the nature and quality of her actions. She understood that her actions would result in the death of a human being; in fact that was the purpose of her actions--to drown her son. Additionally, neither identifies the central issue, which is that without knowing that the act is wrong, the woman could not have formed the requisite criminal intent. Answer choice B is incorrect because it states the test for insanity under the Durham rule.

Congress, without holding hearings or providing public notice, passed a bill immediately signed into law by the President. The bill terminated supplemental security income (SSI) payments to all resident aliens. The reason for the legislation was to address a budgetary shortfall. A resident alien who depended upon the payments to acquire food and shelter filed suit to challenge the constitutionality of the law. How is the court likely to rule on the law's constitutionality?

Uphold it, because Congress has plenary power over aliens. Because Congress has plenary power over aliens under Article I, a federal alienage classification is likely valid unless it is arbitrary and unreasonable. Since the reason for the law (i.e., to reduce the deficit) is rational, it is constitutional. Answer choice B is incorrect because a person's interest in receiving a government benefit is a property interest for which due process must be provided in the denial or termination of that benefit. However, Congress still had the authority to enact this law based on its plenary power over aliens. Answer choice C is incorrect because due process protection does not extend to the process by which a law is enacted. A person does not have a right to notice as to proposed legislation, nor does a person who would be affected by the legislation have a right to be heard. Answer choice D is incorrect because although alienage is a suspect class with regard to action taken by a state government, the statute in question was a federal statute.

A merchant and manufacturer entered into a contract for the sale of a large grandfather clock built by the manufacturer. The manufacturer already had a number of grandfather clocks built, each with slight aesthetic differences, and the contract provided for the sale of a particular clock identified by serial number. The contract specified that the manufacturer would ship the clock by a third-party carrier. However, the contract did not specify either who was to pay the costs of carriage or the place of tender for the clock. The manufacturer shipped the clock identified in the contract to the merchant via a carrier pursuant to a proper contract for its carriage. The carrier delivered the clock to the merchant and the merchant verified that the clock delivered was the one specified in the contract. Under these facts, when did risk of loss pass to the merchant?

When the clock was delivered to the carrier and a proper contract for its carriage was made. Under UCC § 2-509(1), a contract that requires the seller to ship goods to the buyer by a third-party carrier is either a shipment or a destination contract. When the contract requires shipment by a third-party carrier, a shipment contract is presumed unless the contract indicates otherwise (i.e., unless it specifies delivery to a particular place). Because this is presumed to be a shipment contract, the risk of loss would pass from the manufacturer to the merchant when the manufacturer duly delivered the clock to the third-party carrier. Therefore, answer choice B is correct. Answer choice A is incorrect because UCC § 2-509(1)(a) and (b) do not pass the risk of loss from the seller to the buyer at the time of contract formation. Answer choice C is incorrect because this is not a destination contract; the contract did not specify delivery to a particular place. Answer choice D is incorrect because, although UCC § 2-510 places the risk of their loss on the seller when a tender or delivery of goods so fails to conform to the contract as to give a right of rejection, since the manufacturer shipped the clock identified in the contract to the merchant, the merchant's act of identifying the clock as conforming to the contract does not alter the passing of risk to the merchant when the clock was delivered to the carrier and proper contract for its carriage was made.

A woman discovered that her boyfriend was cheating on her. She approached the bartender at her local bar, who she had heard was a member of a biker gang, and offered him $500 to beat up her boyfriend. The bartender agreed immediately. The bartender was actually an undercover officer, however, and he did not intend to beat up the boyfriend. The woman was arrested the next day and charged with solicitation, conspiracy, and assault. The jurisdiction recognizes the common law regarding these crimes. Are the charges against the woman proper?

Yes as to solicitation and assault, but no as to conspiracy. Solicitation is the enticing, encouraging, requesting, or commanding of another person to commit a crime with the intent that the other person commit the crime. The woman was guilty of solicitation when she encouraged the bartender to beat up her boyfriend. It is irrelevant that the bartender never intended to commit the crime. Assault is an attempt to commit a battery (the unlawful application of force to the person of another, which can be applied through a third person acting under the defendant's direction). Attempt requires a substantial step toward the commission of a crime, such as solicitation of an innocent agent to engage in criminal conduct, coupled with intent to commit that crime. Both elements are satisfied in this case. Thus, the woman may be charged with both solicitation and assault. Answer choice A is incorrect because the elements of assault are satisfied. Answer choices C is incorrect because there was no agreement, and thus no conspiracy. Conspiracy requires an agreement between two or more persons. When only one party has the intent to agree, there is no conspiracy. In this case, the bartender never intended to agree, and thus there was no conspiracy. Answer choice D is incorrect because the woman can be properly charged with both solicitation and assault.

Two police officers responded to a call about a domestic disturbance. When they arrived at the home in question, they found a man and woman screaming at each other. One officer handcuffed the man and took him outside, while the other officer spoke with the woman inside the house. Without reading the man his Miranda rights, the officer asked the man to explain what happened. The man stated that the woman had tried to find the gun he had hidden in the bedroom closet. The officer retrieved the gun. The man, who was a convicted felon, was later charged with possession of a firearm as a felon. At trial, the man moved to suppress his statement about the gun and the gun itself, arguing that he did not receive Miranda warnings prior to being questioned by the officer. Is the defendant likely to succeed in having the evidence suppressed for Fifth Amendment reasons?

Yes, as to the statement only. Any statement obtained as the result of a custodial interrogation may not be used against the suspect at a subsequent trial unless the police informed the suspect of his Miranda rights. However, the United States Supreme Court has ruled that derivative physical evidence (e.g., a gun) that has been obtained as a result of a voluntary, uncoerced confession that itself is inadmissible due to the failure by police to give Miranda warnings is admissible. In this case, the man's statement was obtained during a custodial interrogation, and the man was not provided with Miranda warnings. Thus, the man's statement about hiding a gun in the closet is inadmissible. Nevertheless, the gun that was found as a result of that statement is admissible despite being derivative physical evidence. Answer choice A is incorrect because the man's statement is likely to be suppressed because he was not provided with Miranda warnings. Answer choices C and D are incorrect because the gun itself is admissible, despite being derivative physical evidence of a tainted confession.

A football player was very angry with his friend. While the two men were in a crowded casino elevator, the friend said that he did not think the football player should continue gambling with his hard-earned money, but should instead save it to pay for his children's future college tuitions. Angered that his friend wasn't minding his own business, the football player swung his arm back to strike him. However, at the second he would have made contact, the elevator lurched, and he hit another man in the elevator instead. The man sustained a concussion. If the football player is charged with battery on the man in a jurisdiction that follows the Model Penal Code, will he be convicted?

Yes, because he intended to strike his friend. Battery is the unlawful application of force to another person that causes bodily harm to that person. When a defendant acts with the intent to cause harm to one person or object and that act directly results in harm to another person or object, the defendant can be liable for the harm caused under the doctrine of transferred intent. In this case, because he intended to strike his friend, the football player can be liable for striking the man instead under the doctrine of transferred intent. Answer choice B is incorrect. The Model Penal Code recognizes liability when purposely or knowingly causing a particular result is an element of an offense. Accordingly, it is not the football player's reckless action in swinging his arm where others could be hit, but his intending to strike his friend that would make him liable for battery on the man. Answer choice C is incorrect. Battery is not a specific intent crime. Because battery is one of the crimes to which the doctrine of transferred intent generally applies, the football players intended actions towards the friend satisfy the intent requirement for battery on the man. Answer choice D is incorrect because heat of passion is a concept that applies to voluntary manslaughter, not battery.

Pursuant to a federal statute, a taxpayer filed a complaint in federal district court against the United States for civil damages. The claim arose from an IRS employee's alleged reckless and intentional disregard of the U.S. Tax Code and related regulations in attempting to collect federal taxes from the taxpayer. The complaint and summons were personally delivered by a 25-year-old relative of the taxpayer to the U.S. attorney for the district in which the action has been filed. The U.S. attorney has challenged the service of process as insufficient. Should the court find that the service was insufficient?

Yes, because process was not also served on the U.S. Attorney General. Answer choice D is correct. When the United States is a defendant in a civil action, service must be made on the U.S. Attorney General as well as the U.S. attorney for the district in which the action has been filed. A party must send a copy of the summons and complaint by registered or certified mail to the U.S. Attorney General. In this case, although the taxpayer's relative properly served the U.S. attorney, the relative failed to mail a copy of the summons and complaint to the U.S. Attorney General by registered or certified mail. Therefore, service of process was insufficient. Answer choice A is incorrect because the mere fact that the taxpayer's relative, as a nonparty who is at least 18 years old, was entitled to serve process is not enough to make his attempted service sufficient. Answer choice B is incorrect. Although service must be made on the U.S. attorney for the district in which an action against the United States has been filed, service must also be made on the U.S. Attorney General. Answer choice C is incorrect. While the complaint is based on actions taken by an IRS employee, the action was filed against the United States only. Consequently, neither the IRS nor its employee must be served.

A state senator was on a committee reviewing the compliance of all state government buildings with federal wheelchair accessibility policies. The senator headed the committee, and as such, delivered multiple speeches during committee meetings about the importance of having not only all government buildings, but also private buildings, comply with the federal wheelchair guidelines regarding accessibility. One day as he was leaving his office, a reporter asked the senator about his views on wheelchair accessibility. The senator repeated verbatim a speech he gave during one of his committee meetings, including a comment that, unless protected by immunity or privilege, would expose him to liability for defamation of another senator. Can the senator be subject to civil liability for the statements he made to the reporter?

Yes, because the Speech or Debate Clause does not apply to state legislators. State legislators are immune from liability for actions within the sphere of legitimate legislative activity. Here, the senator's comments made during committee meetings were within the sphere of legitimate legislative activity. However, the comments he made to the reporter, despite the fact that they were verbatim of what he said in a committee meeting, are not. Thus, he is not immune from civil liability arising from his comments. Answer choice A is incorrect. The Speech or Debate Clause does not apply to state legislators, but under the principles of federalism, state legislators are immune from liability for actions within the sphere of legitimate legislative activity. Therefore, the inapplicability of the Speech and Debate Clause is not determinative here. Answer choice C is incorrect because the Speech and Debate Clause does not apply to state senators. Answer choice D is incorrect. The state senator would only be immune from liability due to the principles of federalism if his actions were within the sphere of legitimate legislative activity, which in this case they were not.

A defendant was tried for theft of a motor vehicle and found guilty. As permitted by state law as a matter of right, he sought to appeal his conviction. His attorney, whom the defendant had hired, timely filed a notice of appeal. However, the appellate court dismissed the appeal due to his attorney's failure to file the necessary supporting documentation required by state law. The defendant subsequently challenged the action of the appellate court on the grounds of ineffective assistance of counsel. Will the defendant's challenge likely be successful?

Yes, because the attorney's unreasonable conduct prevented the defendant from pursuing his appeal. There is a two-part test for establishing ineffective assistance of counsel: (i) the representation of a defendant by the defendant's attorney must fall below an objective standard of reasonableness, and (ii) the attorney's deficient performance prejudiced the defendant. Here, although the attorney took the necessary first step of filing a notice of appeal, the attorney unreasonably failed to take the second step of filing the necessary document. As a consequence, the defendant was prejudiced (i.e., his appeal was dismissed without a hearing). Answer choice A is incorrect because, although the defendant's attorney did timely file a notice of appeal, the attorney's failure to file the documentation necessary to pursue the appeal was unreasonable. Answer choice B is incorrect because, although there is a presumption that an attorney's representation of a client was constitutionally adequate, this presumption may be overcome. Answer choice C is incorrect because the protection against ineffective assistance of counsel extends to an attorney hired by a defendant as well as to court-appointed counsel.

A highway was known as a drug corridor used to transport drugs between two cities. The drug unit of a state police force launched an initiative to combat drug trafficking along this section of highway. They formulated a plan to conduct checkpoint stops to check for drugs along certain sections of the highway. Under the plan, the police would stop every hundredth car that passed the checkpoint, and a trained canine would sniff for the presence of drugs. The defendant was stopped at a checkpoint, and the canine detected the presence of cocaine in the trunk of his car. The police then searched the trunk, where they found large amounts of cocaine. The entire stop lasted less than three minutes. The defendant was charged and tried for drug trafficking crimes. At his trial, he moved to suppress evidence of the drugs found during the search. Are the drugs likely to be suppressed?

Yes, because the checkpoint stop constituted an unreasonable seizure. Police may stop an automobile at a checkpoint without reasonable, individualized suspicion of a violation of the law if the stop is based on neutral, articulable standards and its purpose is closely related to an issue affecting automobiles. A roadblock to perform sobriety checks has been upheld, while a similar roadblock to perform drug checks has not. Additionally, absent reasonable suspicion, police extension of a permissible traffic stop in order to conduct a dog sniff violates the Fourth Amendment's protection against unreasonable seizures. In this case, the checkpoint stops would constitute an unreasonable seizure because their purpose was to only to perform drug checks rather than to prevent an issue affecting automobiles (for example, driving safely). Answer choice A is incorrect because although the sniff provided probable cause for the search of the trunk, the evidence seized would not be admissible because the stop itself violated the Fourth Amendment. Answer choice B is incorrect because, even if the standards are neutral and articulable, the purpose of the stop must be permissible. Checkpoint stops to perform drug tests are not permissible. Answer choice D is incorrect because the use of a trained dog to sniff for the presence of drugs does not violate the reasonable expectation of privacy present during a valid stop. Thus, a canine search may be performed without probable cause, provided the stop itself meets other constitutional requirements.

A plaintiff filed a claim against a defendant, a corporation incorporated in State A in a federal district court located in State C. The complaint alleged a violation of the federal antitrust statute, which contains a provision for nationwide service of process, but did not specify any amount in controversy. The complaint asserted that the antitrust violation primarily occurred at the corporation's principal place of business, which is located in State B. Can the federal district court in State C exercise personal jurisdiction over the defendant?

Yes, because the federal antitrust statute provides for nationwide service of process. A statute can contain a provision for nationwide service of process to attain personal jurisdiction. While a federal court generally is not vested with nationwide personal jurisdiction, a federal court does have national personal jurisdiction when authorized by federal statute. Here, the federal antitrust statute contains a provision for nationwide service of process. Accordingly, the federal district court in State C can properly exercise personal jurisdiction over the defendant based on service of process pursuant to the provision in the federal antitrust statute. Answer choice A is incorrect because federal question cases, such as cases brought under a federal statute such as this federal antitrust statute, have no amount-in-controversy requirement. Answer choice B is incorrect. Although the defendant does not have sufficient contacts with State C, the provision in the federal antitrust statute authorizes the exercise of personal jurisdiction over the defendant by the federal district court in State C. Answer choice C is incorrect because even though a defendant corporation may have more than one state of citizenship, they are not automatically subject to personal jurisdiction anywhere.

An undercover officer infiltrated a gang by posing as a gang member. While he was working undercover, the officer heard that a member of the gang had shot and killed one of the leaders of a rival gang. The next time the officer and the gang member suspected of the murder were alone, the officer brought up the shooting, indicating that he was very impressed by whoever had the courage to commit such an act. The gang member, taking the bait, bragged that he had shot the rival gang leader, but told the officer to keep the information secret. The gang member was arrested for the murder and the prosecution sought to introduce the statement made to the undercover officer. Is the statement likely to be admitted?

Yes, because the gang member did not know that the undercover officer was a police officer. Miranda warnings are not required if the suspect being questioned is not aware that the interrogator is a police officer. Thus, an undercover officer may question a suspect without informing him of his rights, as was done in this case. Note, too, that Miranda warnings only apply to custodial interrogations, and in this case, the gang member was not in custody. Answer choice A is incorrect because the suspect did not know the undercover officer was a police officer, so Miranda warnings were not required. Answer choice B is incorrect because deceit or fraud by an interrogator (e.g., lying about a co-conspirator's confession) does not itself make a confession involuntary. In this case, the officer's deception regarding his identity did not make the confession involuntary. Answer choice D is incorrect because interrogation refers not only to express questioning, but also to any words or actions that the police know or should know are likely to elicit an incriminating response. In this case, the officer made a statement that he knew was likely to elicit an incriminating response. While the statement was arguably an interrogation, it was not a custodial interrogation. Accordingly, answer choice C is a better answer.

A man who worked nights was unable to sleep during the day because of the persistent barking of a neighbor's dog. Despite repeated requests to the neighbor to address the problem, the barking persisted. The man decided to purchase a pistol to kill the dog. Unfamiliar with firearms, the man bought a pistol, but accidentally bought blank ammunition. Standing in his own yard, the man fired several shots at the barking dog that was about 10 feet away in his neighbor's yard, thinking he was firing real bullets. The dog was unharmed because the blanks were incapable of inflicting harm. In the applicable jurisdiction, malicious destruction of property is a statutory crime. The highest court of this jurisdiction has ruled that this statutory crime requires the reckless destruction, injury, or defacement of the property of another. Is the man likely to be found guilty of attempted malicious destruction of property?

Yes, because the man was unaware that the blank bullets would not harm the dog. The crime of attempt requires that the defendant act with the specific intent to commit the attempted crime. Here, because the man was unaware that the blank bullets would not harm his neighbor's dog, the man possessed the necessary specific intent to commit the crime of attempted malicious destruction of property. Answer choice A is incorrect. In determining whether a defendant possesses the necessary mens rea, it is the defendant's subjective mental state that is relevant, not the mental state of a reasonable person in the defendant's situation. Answer choice B is incorrect because impossibility is not a defense to an attempt charge if the crime attempted is factually impossible to commit due to circumstances unknown to the defendant. Here, the man's lack of awareness that his ammunition would prevent him from harming the dog does not serve as a defense to the crime of attempted malicious destruction of property. Answer choice C is incorrect. Although the crime of malicious destruction of property requires the defendant to act recklessly, attempt requires the defendant to act with the specific intent to commit the attempted crime. Therefore, the man's recklessness is not determinative of his guilt for attempted malicious destruction of property.

An officer pulled over the defendant for speeding. When he ran the defendant's plates, the officer saw that there was an outstanding warrant for the defendant's arrest based on her failure to pay child support. The officer arrested the defendant and placed her in the backseat of his squad car. The officer then returned to the defendant's car and saw that her purse was sitting on the passenger seat. The officer searched the purse and found a small amount of marijuana. The defendant was later charged with drug possession. She has moved to suppress evidence of the marijuana. Is the defendant likely to succeed in having evidence of the marijuana suppressed?

Yes, because the officer did not have a reasonable belief that the passenger compartment of the vehicle contained evidence of the offense. In order to justify a warrantless search of an automobile incident to arrest, the Fourth Amendment requires that law enforcement demonstrate either (i) that the arrestee is within reaching distance of the passenger compartment at the time of the search and, as a result, may pose an actual and continuing threat to the officer's safety or a need to preserve evidence from being tampered with by the arrestee or (ii) that it is reasonable that evidence of the offense of arrest might be found in the vehicle. In this case, the defendant was not within reaching distance of the passenger compartment of the car, and it was not reasonable that evidence of the violation of a child support order would be found in the car. Accordingly, the officer could not search the purse without a warrant. Answer choice A is incorrect because, although an officer may search an individual and containers "immediately associated" with the individual pursuant to a lawful arrest, in this case the purse was in the vehicle and not immediately associated with the defendant. Answer choice B is incorrect because, although the Belton rule previously provided that an officer could conduct a full search of a vehicle pursuant to a lawful arrest, the Supreme Court has since limited the parameters of such a search as previously described. Answer choice D is incorrect because an officer is not necessarily required to have probable cause to search a vehicle incident to a lawful arrest. Accordingly, answer choice C is a better answer. [Note: "Passenger compartment" is the phrase used by the U.S. Supreme Court to describe the area of a vehicle in which passengers ride, to be distinguished from the trunk, for example.]

A police officer spotted a disheveled middle-aged male sitting on a bench in a public park. The officer did not recognize the man, but was aware that the park was frequently used as a site for illegal drug transactions. The officer approached the man who remained seated on the bench. The officer asked the man for his name. When the man refused, the officer arrested him. Under state law, it is a misdemeanor for an individual to intentionally refuse to give his name to a police officer who, having lawfully detained the person, has requested it. Although a search of the man revealed that he did not possess any drugs, he was charged with violating the law and fined. The man challenged his conviction as a violation of both the federal and state constitutions. The highest court in the state upheld the conviction as valid under both. The man sought review of his conviction in federal court on the grounds that it violated his federal constitutional rights. Should the federal court rule in favor of the man?

Yes, because the officer lacked reasonable suspicion to detain the man. The officer, at the time the officer approached the man, lacked reasonable suspicion that the man had committed a crime. As a consequence, under the Fourth Amendment, the encounter was not a valid Terry stop. Therefore, by the terms of the statute, the man was not lawfully detained. Answer choice A is incorrect. The state's highest court did not overturn the man's conviction on adequate and independent state grounds, but upheld it. The state court, by upholding the conviction, at least tacitly determined that the officer had lawfully detained the man. Such a ruling violated the man's Fourth Amendment right against unreasonable searches and seizures because the officer lacked reasonable suspicion to detain the man. Answer choice B is incorrect because a statute that requires a person to identify himself does not violate his Fifth Amendment right against self-incrimination, at least where the person does not have a reasonable belief that, by doing so, the person would be incriminating himself. Answer choice D is incorrect because it is irrelevant. At issue was whether the police officer had reasonable suspicion that the man possessed illegal drugs, not whether the man actually did. In addition, the man was not charged with drug possession, but with refusing to identify himself.

A city enacted ordinances that regulated new construction within a community on the edge of the city but still within the city's boundaries. The ordinances required that any new construction within a ten-block radius of the main street in the community must include retail establishments on the first level. Based on multiple studies, the city knew that current city-dwellers lived in the city for the outstanding amenities, including entertainment and dining establishments. The same studies also revealed that current city-dwellers were dissatisfied with severe traffic problems and overcrowding within the city. The ordinances were enacted by the city to encourage current city-dwellers to relocate to the community by creating a commercial district in the community without the challenges of overcrowding and traffic. After enacting a similar statute on a much smaller scale, the city had seen a decrease in city traffic and overcrowding and knew it could replicate these results on a larger scale. The head of a construction company files suit, alleging that the ordinance prevents him from being able to construct a planned apartment complex in accordance with its current design. The city defends the application of the ordinances on the basis that busier areas, including commercial districts, are generally much safer. Is the city's defense likely to be successful?

Yes, because the ordinance is related to a legitimate state interest. Most legislation related to zoning is reviewed under the rational basis standard, and a law meets that standard if it is rationally related to a legitimate state interest. Laws are presumed valid under this standard, so the burden is on the challenger to overcome this presumption by establishing that the law is arbitrary or irrational. In court, the government's stated interest in enacting the law need not be one that it offered when the law was passed, and here, the city's stated interest constitutes a legitimate interest. The ordinances, while not necessarily the most direct way to accomplish that interest, are rationally related to it. Answer choice A is incorrect because the city does need to meet the rational basis standard when enacting laws related to zoning, though the burden falls on the challenger to demonstrate that the law is not rationally related to the interest. Answer choice C is incorrect because the government interest need not be stated when the law was passed, as long as the stated interest is legitimate. Answer choice D is incorrect because the zoning ordinance does not violate the company's due process rights because it passes the rational basis test.

In response to unusual earthquake activity, a large city enacted an ordinance that required the owner of any habitable structure of more than two stories to permit the permanent placement of discrete and self-contained seismographic equipment owned by the city on the lowest floor of the structure. The purpose of this ordinance was to create an early detection network for seismic activity. The ordinance contained no provision for compensation of an owner of a structure subject to this ordinance because the space required for the equipment was minimal. Knowing that the city had enacted this regulation, a company purchased a tract of land and began construction of a three-story apartment building. The company sued, claiming that this ordinance violated its constitutional rights. Should the court rule in favor of the company?

Yes, because the ordinance results in a permanent physical occupation of the property. The Due Process Clause of the Fourteenth Amendment incorporates the Takings Clause of the Fifth Amendment, thereby making it applicable to the states. A taking has occurred when the governmental regulation results in a permanent physical occupation of the property. In this case, the placement of the seismographic equipment in the building would result in the permanent physical occupation of the building by the city. Consequently, this placement would constitute a taking. Because the company would not be compensated for the taking, it is unconstitutional. Answer choice A is incorrect because a person who acquires property rights after the adoption of a regulation that affects those rights may nevertheless challenge the regulation as an unconstitutional taking. Answer choice B is incorrect because, even though the ordinance would have a minimal economic impact on the company because the equipment itself would occupy only a minimal space on the lowest floor of the apartment building, the ordinance mandates the permanent physical occupation of the property by the government's seismic-detection equipment. Answer choice C is incorrect because the ordinance is not an exaction. An exaction occurs when a local government exacts promises from a developer, such as setting aside a portion of the land being developed for a park, in exchange for issuing the necessary construction permits. Moreover, an exaction does not necessarily constitute an unconstitutional taking.

After a college professor discovered that students were submitting essays purchased online as their own work, the college implemented new school-wide procedures to identify and penalize plagiarism by students. The procedures mandate that students submit lengthy assignments through an online service that requires an expensive monthly subscription. A student on the college paper published the following later that week: "It is ironic that this costly choice comes as a reaction to reports of plagiarism from a professor who, in his days at this institution, attempted to cheat and lie his way to a degree." The professor has sued the student for libel, and the student alleges truth as a defense. After the professor testified as to his character for honesty, the student seeks to admit a disciplinary report indicating that the professor failed a political science class for bringing commercially prepared material to use on the final exam in violation of the honor code. Should the court admit the disciplinary report as substantive evidence?

Yes, because the report is a means of proving an essential element of a defense. Character evidence is admissible as substantive evidence when character is an essential element of a claim or defense, rather than a means of proving a person's conduct. Here, the truth of the student's allegations is an affirmative defense to the claim of libel, and the student may offer specific instances of the professor's conduct to support that affirmative defense. Answer choice B is incorrect because, although a criminal defendant can "open the door" to attacks on his character, this rule does not apply in civil cases. Answer choice C is incorrect because the evidence can be admitted as substantive evidence to support the student's affirmative defense. Answer choice D is incorrect because, although there are a number of limitations on the use of specific instances of conduct to prove character, when character evidence is admissible as evidence in a civil case (i.e., evidence that is an essential element of a claim or defense), it may be proved by specific instances of a person's conduct.

The police had accumulated reliable information that a doctor was illegally dealing prescription drugs. One day, the police received a reliable tip that a stainless steel suitcase containing stolen prescription drugs would be delivered to the doctor's home the following afternoon. The police obtained a search warrant to search for and seize the suitcase when it arrived at the doctor's home the next day. The police immediately put the doctor's home under surveillance. About five minutes after a woman dropped off a stainless steel suitcase at the home, the police knocked on the doctor's door. The doctor was given the search warrant immediately upon opening the door, and the police found the suitcase unopened on the kitchen table. After confirming that the suitcase contained prescription drugs, they seized the suitcase and put handcuffs on the doctor. The police then fanned out through the house, looking in every room and closet. They found no one else, but one officer found an automatic weapon in a box on a closet shelf in the doctor's bedroom. In addition to charges relating to dealing prescription drugs, the doctor was charged with unlawful possession of weapons. The doctor moved to suppress the automatic weapon. Should the court grant the motion to suppress the automatic weapon?

Yes, because the search exceeded the authority granted by the warrant. The warrant was valid, but its validity was triggered by and limited to the delivered suitcase. Accordingly, once the only object of that search was discovered, the warrant did not authorize a further exploratory search of the house. Answer choice A is incorrect because probable cause is not a sufficient reason to conduct a warrantless search of the house beyond the authorized "protective sweep." Answer choice B is incorrect because officers may conduct a "protective sweep" of a home only if they have reason to believe that others inside the home may pose a danger to them. Even if the facts in this scenario justified such a protective sweep, such a sweep is limited to a cursory visual inspection of those places in which a person might be hiding and would not have extended to the box on a closet shelf. Answer choice D is incorrect because the probable cause requirement of a warrant is satisfied when, at the time that the warrant is issued, there is probable cause to believe that the triggering condition will occur and, if that condition does occur, there is a fair probability that contraband or evidence of a crime will be found in a particular place.

Motivated by a desire to prevent animal cruelty, activists frequently broke into facilities of companies that conduct animal testing and released the animals. These break-ins usually occurred after a certain animal-rights group organized a picket against these companies. Wanting to discourage this type of criminal activism, the city enacted an ordinance stating that no person was permitted to picket against animal testing in any public forum. The animal-rights group wanted to organize a picketing protest against the use of animals in testing beauty products on the sidewalk in front of a particular facility. After their arrest for violating the new ordinance, the animal-rights group challenges the constitutionality of the ordinance based upon their First Amendment right to free speech. Assuming no federal law is applicable and the group has standing, are they likely to succeed in their challenge?

Yes, because the sidewalk is a traditionally public forum. In traditionally public forums, like sidewalks, the government may impose reasonable restrictions on the time, place, or manner of protected speech, provided the restrictions are content-neutral as to both subject matter and viewpoint, are narrowly tailored to serve a significant governmental interest, and leave open ample alternative channels for communication of the information. Although the city may have an important interest in preventing the breaking and entering into the animal testing facilities, the ordinance preventing all picketing on the subject of animal testing is not narrowly tailored to further this interest. Rather, the regulation suppresses all picketing on this subject. Additionally, this regulation is not content-neutral, since it would appear to permit picketing about other issues on the same sidewalk. Answer choice A is incorrect because furthering a governmental interest does not justify a content-based restriction of speech in a public forum. The ordinance forbids all picketing on this subject matter, making no attempt to limit its restriction to picketing that could reasonably incite criminal activity or violence. Therefore, regardless of the intended purpose, the ordinance impermissibly limits protected speech and is unconstitutional. Answer choice B is incorrect because, in addition to permitting alternative channels of speech, a limitation on protected speech in a public forum must be content-neutral. Thus, this reasoning is insufficient to make this ordinance constitutional. Answer choice C is incorrect because it is a misstatement of the law. Prior restraints on speech may be permitted when there is a particular harm to be avoided and certain procedural safeguards are provided to the speaker.

In order to save money, a state adopted a law restricting voting times and reducing the number of polling sites on Election Day. The law did not significantly impact the ability of voters to cast their ballots. The law applied to all state, local, and federal elections occurring on Election Day. Does Congress have the authority to override this law?

Yes, because the state law regulates federal elections. The Elections Clause explicitly empowers Congress to override state laws concerning federal elections. Here, the legislation attempts to regulate the federal election process by limiting voting times and polling sites as applied to federal elections, and Congress has the power to override such legislation. Answer choice A is incorrect because even if the voters are able to cast their votes, the limitations in the election process apply to federal elections. Congress has authority to override such a regulation. Answer choice B is incorrect because, even though the changes are unrelated to the suppression of ideas, Congress can nevertheless override state laws concerning federal elections. This power is not restricted to state election laws that suppress ideas. Answer choice C is incorrect because Congress does not have blanket authority to override a state election law that applies to elections for state offices.

A defendant is on trial for robbery. The prosecution has alleged that when the defendant went to a man's home for a dinner date, she gave him a glass of wine poisoned with a strong tranquilizer and, when the man passed out, robbed his home. The prosecution wants to call an ambulance technician who arrived on the scene after the man woke up and called for an ambulance. The technician intends to testify that when she found the man, still sitting disoriented on the floor, the man said, "I think that wine my date brought was poisoned, is she ok? I think it made me sick, and she may have had some too, but I don't remember." Is the ambulance technician's testimony admissible?

Yes, because the statement was reasonably pertinent to medical treatment. A statement describing medical history or past or present symptoms is not hearsay if it is made for medical diagnosis or treatment. A statement of the cause or source of the condition is admissible as an exception to the rule against hearsay if it is reasonably pertinent to diagnosis or treatment. Here, the man's statements concerned a possible source of his illness and injury, assisting in the diagnosis and treatment of both himself and, possibly, the defendant. Therefore, the statement is admissible hearsay, regardless of whether the man is available to testify. Answer choice A is incorrect because a statement need not be made to a physician to fall under this hearsay exception. Statements to other medical personnel, including hospital attendants and ambulance drivers, or even to family members, may be included. Answer choice B is incorrect because the exception for statements made for medical diagnosis or treatment does not require the declarant's unavailability. Answer choice C is incorrect. A statement made about a startling event or condition while the declarant is under the stress of excitement that it caused is not excluded as hearsay. Under this exception to the hearsay rule, the event must shock or excite the declarant, and the statement must relate to the event. However, concern for someone's safety because she may have been exposed to a danger at some unknown time in the past is unlikely to elicit the level of shock or excitement that would make the statement reliable enough to fall under this hearsay exception. Therefore, these facts do not appear to support an excited utterance hearsay exception.

A city enacted legislation that required schools to automatically expel any male student who bullied another student, with bullying defined as "physical violence not used in self-defense." The legislation did not reference bullying committed by female students. The law was enacted in response to a high level of teen suicides that were linked to severe bullying, and it directly followed similar legislation in a neighboring state that was successful in the reduction of teen suicides over a five-year period. The in-depth analysis of the reduction of suicides in the neighboring state revealed, in those cases where bullying resulted in suicide, the vast majority involved bullying initiated by males, which was more severe than female bullying and less likely to stop without intervention. Would this statute likely survive a constitutional challenge?

Yes, because the statute is substantially related to preventing teen suicides. Discrimination based on gender is "quasi-suspect" and subject to intermediate scrutiny, which is less stringent than strict scrutiny but tougher than the rational basis test. Just as with suspect classifications and fundamental rights, there must be discriminatory intent by the government to trigger intermediate scrutiny. Even if discriminatory intent can be shown, under intermediate scrutiny, the burden is on the state to show that a statute or regulation that treats the sexes differently is substantially related to an important governmental interest. Here, the city has clear evidence that male-driven bullying is more severe, and that severe bullying was linked to teen suicides. The statute was enacted to address these concerns and not to simply discriminate against male students. Answer choice B is incorrect because the statute applies differently to different genders, so the rational basis standard would not apply. Answer choice C is incorrect because the statute does not need to be the least restrictive means to accomplish the objective, as intermediate scrutiny rather than strict scrutiny applies. Answer choice D is incorrect because although the statute does discriminate based on sex, it does so while meeting the applicable level of scrutiny. Statutes that distinguish between the sexes are not automatically invalid.

An undergraduate student planned to transfer to a university in the town where her aunt resided and to live in an on-campus dorm. She told her aunt about her plans in an email. The next day, the aunt replied with an email containing the following: "I am so excited! If you don't want to deal with a roommate, don't apply for a room in the dorms, you can just stay in my spare room rent-free!" Elated, the student immediately submitted her transfer application without submitting an application for a dorm room on campus. After the deadline to apply for a dorm room had passed, the student's aunt called and explained that she had no room for the student because she was renting out her spare room to another student who was willing to pay rent. The student had to sign a lease in an off-campus apartment and ended up paying much more than she would have paid to live in the dorm. If the student sues her aunt to recover damages, is the student likely to recover?

Yes, because the student decided not to apply for a dorm room in reasonable reliance on her aunt's promise to provide her a room. A promise is binding if the promisor should reasonably expect it to induce action or forbearance on the part of the promisee or a third person, the promise does induce such action or forbearance, and injustice can be avoided only by enforcement of the promise. Here, the student reasonably relied on her aunt's promise of rent-free boarding when she did not apply for on-campus housing. Therefore, the student will likely succeed in her action against her aunt. Answer choice A is incorrect because the student has reasonably relied on this promise to her detriment. Therefore, the promise is enforceable under the doctrine of promissory estoppel. Answer choice B is incorrect because the student did not expressly need to accept the aunt's promise to provide her with housing. The aunt specifically told her not to apply for on-campus housing if she did not want to have a roommate, and promised that she would have a rent-free room if she wanted it. Because the student reasonably relied on this promise to her detriment, the promise is enforceable. Answer choice C is incorrect because these facts are insufficient to create a consideration substitute for an otherwise unenforceable promise.

A woman received a facial rejuvenation treatment. During the treatment, the woman suffered severe damage to her skin due to her dermatologist's gross negligence. The woman filed a federal diversity suit in State A against the dermatologist. The woman lives in State B, but she had been visiting the treatment facility that the dermatologist operates in State A. The contract that the woman signed with the dermatologist permits the recovery of attorney's fees. The woman is seeking damages of $25,000 to recover for the cost of the treatment, $40,000 for pain and suffering, $10,000 for attorney's fees, and $100,000 in punitive damages. Does the woman's claim satisfy the amount-in-controversy requirement?

Yes, because the total amount in controversy is $175,000. In a federal diversity claim, the amount in controversy must exceed the sum or value of $75,000, exclusive of interest, costs, and collateral effects of a judgment. Although interest and costs are excluded from the amount in controversy, attorney's fees may be made part of the amount in controversy if the fees are recoverable by contract or statute. Punitive damages, as well, may be permitted to be made part of the amount in controversy. For this reason, answer choices A and B are incorrect. Answer choice C is incorrect because the amount in controversy must exceed $75,000. A claim for exactly $75,000 will fail the amount-in-controversy requirement. Because punitive damages are included in the amount in controversy, the amount totals $175,000.

A woman was at a Halloween party at a local drinking establishment. A strange man, dressed as the Unabomber, came up to her at the bar and started making lewd comments regarding her costume. The woman rebuffed the man, and the man did not take her rejection graciously. He continued to make vulgar comments to her, and then he lifted up his sweatshirt and showed her a crude-looking homemade bomb strapped to his abdomen. While laughing, he said that if she didn't give him a kiss, he would set off the bomb. The woman told the bartender to call the police. The police arrived with a bomb squad, but it was quickly concluded that the bomb was fake. Can the man be successfully prosecuted for criminal assault?

Yes, because the woman was afraid that the man would set off the bomb.A is correct. Assault is intentionally placing another in apprehension of imminent bodily harm. The victim's apprehension must be reasonable. Here, although it was Halloween and it is normal for people to dress up as unsavory characters, the woman did not know the man. In addition, he was dressed up as the Unabomber, and he continuously made rude and distasteful comments to her. Thus, it was reasonable for the woman to be in apprehension of imminent bodily harm when he threatened to set off the bomb if she refused to kiss him. Answer choice B is incorrect because the "fear of harm" type of assault (also called "apprehension assault") is a general-intent crime—the defendant must intend to cause bodily harm or apprehension of such harm. Answer choice C is incorrect because even if the bomb was a fake, it is still reasonable for the woman to have been in apprehension of an imminent bodily harm as discussed above. Answer choice D is incorrect because neither assault nor battery is a specific-intent crime.

For years, an artist has lived in a house she rents on a month-to-month basis. Five years ago, she got the owner's permission to install a sculpture she had designed for the house. The elaborate wrought iron sculpture was bolted to the walls and woven through the railings of an open spiral staircase connecting the two floors. Recently, the owner told the artist that the monthly rent would be raised by a substantial amount. Because he doubted the artist would be able to pay the increased rent on her irregular income, he informed her that her tenancy would terminate in two months. The owner also told her that she must remove her sculpture within a week because he wanted to show the house to potential tenants and believed the sculpture would make the home harder to rent. Because she was unwilling to destroy the sculpture, the artist was unable to remove the sculpture in that time. However, a potential tenant who toured the home a week later loved the sculpture and told the owner she wanted to rent the home with the sculpture intact. The owner then informed the artist that she was no longer permitted to take the sculpture. By this time, the artist had devised a plan to deconstruct the sculpture and remove it in fragments. Her plan would allow her to remove the sculpture, reconstruct it elsewhere, and leave the house in its original condition. However, her plan will take two days longer than the time remaining on her lease. May the artist legally remove the sculpture in accordance with her plan?

Yes, because there is no agreement that limits her ability to remove the structure. In general, absent an agreement to the contrary, a tenant may remove a fixture that the tenant has attached to the leased property if (i) the leased property can be and is restored to its former condition after the removal, and (ii) the removal and restoration is made within a reasonable time. Here, the artist can remove the sculpture without damaging the home. Although a reasonable time for removal generally does not extend beyond the termination of the lease, it may do so when (i) the termination is not due to a breach by the tenant, and (ii) the date of termination is not foreseeable by the tenant sufficiently far enough in advance to permit removal before the termination of the lease. Here, the owner gave the artist as short a notice of the termination of the lease as permitted by law, and the termination is not due to the artist's breach. Therefore, removing the sculpture within two days of the end of her lease is reasonable, despite the owner's originally unreasonable deadline of one week. Therefore, because no agreement limits her right to remove this fixture, she may do so before she vacates the house. Answer choice A is incorrect. As stated above, there are situations in which a reasonable time for removing a fixture may extend beyond the termination of a lease. Therefore, answer choice A misstates the law. Answer choice B is incorrect because the owner's demand that the artist remove the sculpture in a week does not constitute an agreement that would limit the artist's ability to remove the fixture. Answer choice C is incorrect because, although the owner's permission to install the sculpture is relevant in determining whether the sculpture constituted waste, it is not relevant to the artist's right to remove the fixture.

After a trademarked cartoon character was almost elected as mayor of a small town as a write-in candidate, the state in which the town was located enacted a statute banning all write-in candidates. Instead, candidates must obtain a set number of signatures on a petition to appear on a ballot. The petition requirement has allowed some independent candidates to appear on ballots in the state, but not all candidates who set out to achieve the required number of signatures have succeeded. A candidate has brought an action challenging the constitutionality of the new rules. Are the state's new rules regarding candidates constitutional?

Yes, because there is no fundamental right to have one's name on an election ballot. There is no fundamental right to have one's name on an election ballot or to hold office through election or appointment. A state may ban all write-in candidates in both primary and general elections, as long as the state provides other reasonable means by which a candidate can get on the ballot. For example, a candidate for elected public office can be required to obtain the signatures of voters on a petition in order to appear on the ballot, as long as the requirement does not deny independent candidates ballot access. Therefore, on these facts, the ban is likely constitutional because the petition signature requirement is a reasonable alternative that allows a candidate to get the candidate's name on the ballot. Answer choice A is incorrect because a person's right to vote does not extend to the right to vote for any possible candidate. Answer choice B is incorrect because the petition signature requirement is not unconstitutional unless it denies independent candidates ballot access or imposes undue burdens on placing new or small parties on the state ballots. This does not appear to be the case here. Therefore, the petition requirement is likely a reasonable alternative to permitting write-in candidates. Answer choice D is incorrect because, although this is a correct statement, a state may only ban all write-in candidates if the state provides other reasonable means by which a candidate can get on the ballot. Therefore, the reason that the rules are likely constitutional is because a reasonable alternative of ballot access exists, not because there is no fundamental right to have one's name on an election ballot

After extensive safety testing, a manufacturer of tempered glass products created a new line of tempered glass bowls that were microwave safe. The bowls were designed to be heated safely for up to six minutes in microwaves that fell within the range of 700 to 1,200 watts in power. These limitations were clearly stated on the packaging of the bowls. The manufacturer sold its bowls to a cookware supplier. The supplier inspected each of the bowls before putting them out for sale. The supplier subsequently sold one of the bowls to a food truck cook. On the first day he had it, the cook put meat in the bowl and placed it into a refrigerator overnight. The next day, the cook took the very cold bowl of meat out of the refrigerator and microwaved it in a 1,000 watt microwave for three minutes. The bowl exploded after two minutes, causing the microwave door to burst open and shards of glass to injure the cook. The cook sued the supplier under a strict products liability theory for failure to warn. At trial, it was established that although the bowls could withstand six minutes in microwaves that fell within the range of 700 to 1,200 watts in power, the bowls could not safely withstand extreme changes in temperature over a short period of time. Will the cook prevail in his suit against the supplier?

Yes, because there was no warning on the packaging that the bowls could not be subjected to extreme fluctuations in temperature. A failure to warn defect exists if there were foreseeable risks of harm, not obvious to an ordinary user of the product, which could have been reduced or avoided by providing reasonable instructions or warnings. For the action to succeed, the failure to include the instructions or warnings must render the product not reasonably safe. Here, there was a foreseeable risk of the bowls bursting because it is foreseeable that someone would take a bowl directly from the fridge and put it into the microwave. However, this risk would not be obvious to an ordinary user of the bowl, and could have easily been avoided if a warning as to this issue had also been added to the bowl's packaging. Answer choice B is incorrect. To bring a strict-liability action, a plaintiff is not required to be in privity of contract with the defendant. Anyone foreseeably injured by a defective product or whose property is harmed by the product may bring a strict-liability action. Appropriate plaintiffs include not only purchasers, but also other users of the product. Here, the cook could have recovered from the supplier regardless of whether he purchased the bowl from the supplier because the cook was injured when using the bowl. Answer choice C is incorrect because, as a commercial supplier of the bowl, the cookware supplier is also strictly liable for personal injury caused by a defective product it sold. Answer choice D is incorrect. Under a strict liability theory, the reasonable care exercised by the supplier in inspecting the bowls prior to sale is not relevant; it would only be relevant if the cook had sued the supplier under a negligence theory.

Based on a reliable insider tip, a reporter investigated the testing of a corporation's salon products on animals. During the course of her investigation, the reporter snuck into the corporate headquarters where the reporter discovered and copied many documents proving the management's efforts to conceal the corporation's animal testing from the public. The reporter's newspaper published several of her stories, resulting in a drastic drop in sales of the corporation's product. The corporation filed a lawsuit against both the reporter for trespass and the newspaper for publishing illegally obtained facts. Can the corporation recover damages in its suit?

Yes, the corporation can recover from either the newspaper or the reporter. Answer choice C is correct because the press has no greater First Amendment rights than does the general public. There is no special privilege allowing the press to invade the rights of others. Members of the press are not immune from the application of generally applicable laws, even if the application of such laws has a negative incidental effect on the ability to gather and report the news. The First Amendment shields the media from liability for publishing information that was obtained illegally by a third party as long as the information involves a matter of public concern and the publisher neither obtained it unlawfully nor knows who did. Here, the newspaper likely knew the information was not lawfully obtained and certainly knows who obtained the information. Therefore, the corporation can recover against both the reporter and the newspaper. Answer choice D is incorrect because this protection does not apply where the media or its employee illegally obtains the information. Answer choice A is incorrect because the newspaper would also be liable to the corporation for publishing facts about the corporation's activities when aware of the source of the illegally obtained information. Answer choice B is incorrect because the reporter would also be liable for illegally obtaining the information. No media protection would be granted to her based on the method of obtaining the information.


Ensembles d'études connexes

Renal Ch2 - Glomerular Physiology

View Set

Automated External Defibrillator for Infants and Children less than 8 Years of Age & Ventilation Techiques

View Set

AP Euro Fall Semester Review - Final, MCQ Unit Progress #1, MCQ Unit Progress #2, ap euro mcq3, euro 3, AP EURO Exam: French Revolution, Ap European history Unit 6 Progress check

View Set

Chapter 6: lEADERSHIP and global organizations

View Set

Semester 1 Physics Final Study Guide

View Set